CardioNerds Decipher The Guidelines Logo

Decipher the Guidelines 2021 ESC Guidelines on Cardiovascular Disease Prevention

Decipher the Guidelines - 2021 ESC CV Prevention
Click to enlarge

Welcome to the CardioNerds Decipher the Guidelines
2021 ESC Guidelines on Cardiovascular Disease Prevention edition! 

The Decipher the Guidelines using high-impact, board-style, clinical vignette based questions to highlight core concepts relevant to your practice. We will do so by releasing several short bite-sized Pods with one question per episode.

This particular series is built around the 2021 ESC Prevention Guidelines and 2019 ACC/AHA Prevention Guidelines, was developed in collaboration with the ACC Prevention of CVD Section, the NLA, and the PCNA. Its creation reflects the passion and energy of 31 individuals spanning medical students through to lead experts from varied backgrounds including physicians, pharmacists, and nurse practitioners.  

The cases discussed in this podcast series are fictional and designed to highlight key teaching points..

Episodes

The following question refers to Section 4.9 of the 2021 ESC CV Prevention Guidelines. The question is asked by Dr. Christian Faaborg-Andersen, answered first by UCSD fellow Dr. Patrick Azcarate, and then by expert faculty Dr. Melissa Tracy.

Dr. Tracy is a preventive cardiologist, former Director of the Echocardiography Lab, Director of Cardiac Rehabilitation, and solid organ transplant cardiologist at Rush University.

The following question refers to Section 4.7 of the 2021 ESC CV Prevention Guidelines. The question is asked by student Dr. Shivani Reddy, answered first by NP Carol Patrick, and then by expert faculty Dr. Eileen Handberg.

Dr. Handberg is an Adult Nurse Practitioner, Professor of Medicine, and Director of the Cardiovascular Clinical Trials Program in the Division of Cardiovascular Medicine at the University of Florida. She has served as Chair of the Cardiovascular Team Section and the Board of Trustees with the ACC and is the President Elect for the PCNA.

The following question refers to Section 4.5 of the 2021 ESC CV Prevention Guidelines. The question is asked by Dr. Maryam Barkhordarian, answered first by pharmacy resident Dr. Anushka Tandon, and then by expert faculty Dr. Noreen Nazir.

Dr. Nazir is Assistant Professor of Clinical Medicine at the University of Illinois at Chicago, where she is the director of cardiac MRI and the preventive cardiology program.

The following question refers to Section 3.4 of the 2021 ESC CV Prevention Guidelines. The question is asked by student Dr. Adriana Mares, answered first by early career preventive cardiologist Dr. Dipika Gopal, and then by expert faculty Dr. Michael Wesley Milks.

Dr. Milks is a staff cardiologist and assistant professor of clinical medicine at the Ohio State University Wexner Medical Center, where he serves as the Director of Cardiac Rehabilitation and an associate program director of the cardiovascular fellowship. He specializes in preventive cardiology and is a member of the American College of Cardiology’s Cardiovascular Disease Prevention Leadership Council.

The following question refers to Figures 6-8 from Sections 3.2 of the 2021 ESC CV Prevention Guidelines. The question is asked by student Dr. Hirsh Elhence, answered first by Ohio State University Cardiology Fellow Dr. Alli Bigeh, and then by expert faculty Dr. Eugene Yang.

Dr. Yang is Professor of Medicine at the University of Washington where he is also the Medical Director of the Eastside Specialty Center and the co-Director of the Cardiovascular Wellness and Prevention Program. Dr. Yang is former Governor of the ACC Washington Chapter and as well as former Chair of the ACC Prevention of CVD Section.  

The following question refers to Section 6.1 of the 2021 ESC CV Prevention Guidelines. The question is asked by MGH internal medicine resident Dr. Christian Faaborg-Andersen, answered first by UCSD early career preventive cardiologist Dr. Harpreet Bhatia, and then by expert faculty Dr. Eugenia Gianos.

Dr. Gianos specializes in preventive cardiology, lipidology, cardiovascular imaging, and women’s heart disease; she is the Director of Women’s Heart Health at Lenox Hill Hospital and Director of Cardiovascular Prevention for Northwell Health.

The following question refers to Section 5.2 of the 2021 ESC CV Prevention Guidelines. The question is asked by MGH medicine resident Dr. Christian Faaborg-Andersen, answered first by Dr. Jessie Holtzman, and then by expert faculty Dr. Laurence Sperling.

Dr. Laurence Sperling is the Katz Professor in Preventive Cardiology at the Emory University School of Medicine and Founder of Preventive Cardiology at the Emory Clinic. Dr. Sperling was a member of the writing group for the 2018 Cholesterol Guidelines, serves as Co-Chair for the ACC’s Cardiometabolic and Diabetes working group, and is Co-Chair of the WHF Roadmap for Cardiovascular Prevention in Diabetes.

Dr. Laurence Sperling is the Katz Professor in Preventive Cardiology at the Emory University School of Medicine and Founder of Preventive Cardiology at the Emory Clinic. Dr. Sperling was a member of the writing group for the 2018 Cholesterol Guidelines, serves as Co-Chair for the ACC’s Cardiometabolic and Diabetes working group, and is Co-Chair of the WHF Roadmap for Cardiovascular Prevention in Diabetes. 

The following question refers to Section 4.7 and Table 18 of the 2021 ESC CV Prevention Guidelines. The question is asked by CardioNerds Academy Intern Student Dr. Shivani Reddy, answered first by Fellow at Johns Hopkins Dr. Rick Ferraro, and then by expert faculty Dr. Roger Blumenthal.

Dr. Roger Blumenthal is professor of medicine at Johns Hopkins where he is Director of the Ciccarone Center for the Prevention of Cardiovascular Disease. He was instrumental in developing the 2018 ACC/AHA CV Prevention Guidelines. Dr. Blumenthal has also been an incredible mentor to CardioNerds from our earliest days.

The following question refers to Section 4.3 of the 2021 ESC CV Prevention Guidelines. The question is asked by CardioNerds Academy Intern Dr. Maryam Barkhordarian, answered first by medicine resident CardioNerds Academy House Chief Dr. Ahmed Ghoneem, and then by expert faculty Dr. Kim Williams.

Dr. Williams is Chief of the Division of Cardiology and is Professor of Medicine and Cardiology at Rush University Medical Center. He has served as President of ASNC, Chairman of the Board of the Association of Black Cardiologists (ABC, 2008-2010), and President of the American College of Cardiology (ACC, 2015-2016).

The following question refers to Sections 3.3 and 3.4 of the 2021 ESC CV Prevention Guidelines.

The question is asked by CardioNerds Academy Intern student Dr. Adriana Mares, answered first by Brigham & Women’s medicine resident & Director of CardioNerds Internship Dr. Gurleen Kaur, and then by expert faculty Dr. Allison Bailey.

Dr. Bailey is an advanced heart failure and transplant cardiologist at Centennial Heart. She is the editor-in-chief of the American College of Cardiology’s Extended Learning (ACCEL) editorial board and was a member of the writing group for the 2018 American Lipid Guidelines. 

The following question refers to Section 3.2 of the 2021 ESC CV Prevention Guidelines. The question is asked by student Dr. Hirsh Elhence, answered first by Mayo Clinic Fellow Dr. Teodora Donisan, and then by expert faculty Dr. Eugene Yang.

Dr. Yang is professor of medicine of the University of Washington where he is medical director of the Eastside Specialty Center and the co-Director of the Cardiovascular Wellness and Prevention Program. Dr. Yang is former Governor of the ACC Washington Chapter and chair of the ACC Prevention of CVD Section.

The following question refers to Section 6.3 of the 2021 ESC CV Prevention Guidelines. The question is asked by Dr. Christian Faaborg-Andersen, answered first by UCSD cardiology fellow Dr. Harpreet Bhatia, and then by expert faculty Dr. Jaideep Patel.

Dr. Patel recently graduated from Virginia Commonwealth University cardiology fellowship and is now a preventive cardiologist at the Johns Hopkins Hospital.

The following question refers to Section 6.1 of the 2021 ESC CV Prevention Guidelines. The question is asked by Dr. Christian Faaborg-Andersen, answered first by Houston Methodist medicine resident Dr. Najah Khan, and then by expert faculty Dr. Eugenia Gianos.

Dr. Gianos specializes in preventive cardiology, lipidology, cardiovascular imaging, and women’s heart disease; she is the director of Women s Heart Health at Lenox Hill Hospital and director of Cardiovascular Prevention for Northwell Health.

The following question refers to Section 4.6 of the 2021 ESC CV Prevention Guidelines. The question is asked by Student Dr. Shivani Reddy, answered first by Johns Hopkins Cardiology Fellow Dr. Rick Ferraro, and then by expert faculty Dr. Eileen Handberg.

Dr. Handberg is an Adult Nurse Practitioner, Professor of Medicine, and Director of the Cardiovascular Clinical Trials Program in the Division of Cardiovascular Medicine at the University of Florida. She has served as Chair of the Cardiovascular Team Section and the Board of Trustees with the ACC and is the President for the PCNA.

The following question refers to Section 4.4 of the 2021 ESC CV Prevention Guidelines. The question is asked by Dr. Maryam Barkhordarian, answered first by medicine resident Dr. Ahmed Ghoneem, and then by expert faculty Dr. Noreen Nazir.

Dr. Nazir is Assistant Professor of Clinical Medicine at the University of Illinois at Chicago, where she is the director of cardiac MRI and the preventive cardiology program.

The following question refers to Section 3.4 of the 2021 ESC CV Prevention Guidelines. The question is asked by student Dr. Adriana Mares, answered first by Brigham & Women’s medicine intern & Director of CardioNerds Internship Dr. Gurleen Kaur, and then by expert faculty Dr. Michael Wesley Milks.

Dr. Milks is a staff cardiologist and assistant professor of clinical medicine at the Ohio State University Wexner Medical Center where he serves as the Director of Cardiac Rehabilitation and an associate program director of the cardiovascular fellowship. He specializes in preventive cardiology and is a member of the American College of Cardiology’s Cardiovascular Disease Prevention Leadership Council.

The following question refers to Section 3.2 of the 2021 ESC CV Prevention Guidelines. The question is asked by CardioNerds Academy Intern, student Dr. Hirsh Elhence, answered first by Ohio State University Cardiology Fellow Dr. Alli Bigeh, and then by expert faculty Dr. Eugene Yang.

Dr. Yang is professor of medicine of the University of Washington where he is medical director of the Eastside Specialty Center and the co-Director of the Cardiovascular Wellness and Prevention Program. Dr. Yang is former Governor of the ACC Washington Chapter and current chair of the ACC Prevention of CVD Section. 

The CardioNerds Decipher The Guidelines Series for the 2021 ESC CV Prevention Guidelines represents a collaboration with the ACC Prevention of CVD Section, the National Lipid Association, and Preventive Cardiovascular Nurses Association.

The following question refers to Section 6.2 of the 2021 ESC CV Prevention Guidelines. The question is asked by Dr. Christian Faaborg-Andersen, answered first by Houston Methodist medicine resident Dr. Najah Khan, and then by expert faculty Dr. Jaideep Patel.

Dr. Patel recently graduated from Virginia Commonwealth University cardiology fellowship and is now a preventive cardiologist at the Johns Hopkins Hospital.

The following question refers to Section 4.9 of the 2021 ESC CV Prevention Guidelines. The question is asked by Dr. Christian Faaborg-Andersen, answered first by UCSD fellow Dr. Patrick Azcarate, and then by expert faculty Dr. Melissa Tracy.

Dr. Tracy is a preventive cardiologist, former Director of the Echocardiography Lab, Director of Cardiac Rehabilitation, and solid organ transplant cardiologist at Rush University.

The following question refers to Section 4.6 and Figure 13 of the 2021 ESC CV Prevention Guidelines. The question is asked by student doctor Shivani Reddy, answered first by NP Carol Patrick, and then by expert faculty Dr. Roger Blumenthal.

Dr. Roger Blumenthal is professor of medicine at Johns Hopkins where he is Director of the Ciccarone Center for the Prevention of Cardiovascular Disease. He was instrumental in developing the 2018 ACC/AHA CV Prevention Guidelines.

The following question refers to Section 4.3 of the 2021 ESC CV Prevention Guidelines. The question is asked by CardioNerds Academy Intern Dr. Maryam Barkhordarian, answered first by pharmacy resident Dr. Anushka Tandon and then by expert faculty Dr. Kim Williams.

Dr. Williams is Chief of the Division of Cardiology and is Professor of Medicine and Cardiology at Rush University Medical Center. He has served as President of ASNC, Chairman of the Board of the Association of Black Cardiologists (ABC, 2008-2010), and President of the American College of Cardiology (ACC, 2015-2016).

The following question refers to Sections 3.3-3.4 of the 2021 ESC CV Prevention Guidelines. The question is asked by student Dr. Adriana Mares, answered first by early career preventive cardiologist Dr. Dipika Gopal, and then by expert faculty Dr. Allison Bailey.

Dr. Bailey is a cardiologist at Centennial Heart. She is the editor-in-chief of the American College of Cardiology’s Extended Learning (ACCEL) editorial board and was a member of the writing group for the 2018 American Lipid Guidelines. 

The following question refers to Section 3.2 of the 2021 ESC CV Prevention Guidelines. The question is asked by student Dr. Hirsh Elhence, answered first by Mayo Clinic Fellow Dr. Teodora Donisan, and then by expert faculty Dr. Eugene Yang.

Dr. Yang is professor of medicine of the University of Washington where he is medical director of the Eastside Specialty Center and the co-Director of the Cardiovascular Wellness and Prevention Program. Dr. Yang is former Governor of the ACC Washington Chapter and current chair of the ACC Prevention of CVD Section.

The following question refers to Section 4.11 of the 2021 ESC CV Prevention Guidelines. The question is asked by Dr. Christian Faaborg-Andersen, answered first by UCSF resident Dr. Jessie Holtzman, and then by expert faculty Dr. Laurence Sperling.

Dr. Laurence Sperling is the Katz Professor in Preventive Cardiology at the Emory University School of Medicine and Founder of Preventive Cardiology at the Emory Clinic. Dr. Sperling was a member of the writing group for the 2018 Cholesterol Guidelines, serves as Co-Chair for the ACC’s Cardiometabolic and Diabetes working group, and is Co-Chair of the WHF Roadmap for Cardiovascular Prevention in Diabetes.

The following question refers to Section 6.1 of the 2021 ESC CV Prevention Guidelines. The question is asked by Dr. Christian Faaborg-Andersen, answered first by UCSD cardiology fellow Dr. Harpreet Bhatia, and then by expert faculty Dr. Eugenia Gianos.

Dr. Gianos specializes in preventive cardiology, lipidology, cardiovascular imaging, and women’s heart disease; she is the director of the Women’s Heart Program at Lenox Hill Hospital and director of Cardiovascular Prevention for Northwell Health.

The following question refers to Section 4.6 of the  2021 ESC CV Prevention Guidelines. The question is asked by student Dr. Shivani Reddy, answered first by NP Carol Patrick, and then by expert faculty Dr. Eileen Handberg.

Dr. Handberg is an Adult Nurse Practitioner, Professor of Medicine, and Director of the Cardiovascular Clinical Trials Program in the Division of Cardiovascular Medicine at the University of Florida. She has served as Chair of the Cardiovascular Team Section and the Board of Trustees with the ACC and is the President Elect for the PCNA.

The following question refers to Section 4.3 of the 2021 ESC CV Prevention Guidelines. The question is asked by Dr. Maryam Barkhordarian, answered first by pharmacy resident Dr. Anushka Tandon, and then by expert faculty Dr. Noreen Nazir.

Dr. Noreen Nazir is Assistant Professor of Clinical Medicine at the University of Illinois at Chicago, where she is the director of cardiac MRI and the preventive cardiology program.

This question refers to Sections 3.1 of the 2021 ESC CV Prevention Guidelines. The question is asked by CardioNerds Academy Intern, student Dr. Hirsh Elhence, answered first by internal medicine resident at Beaumont Hospital and soon to be Mayo Clinic cardiology fellow and Dr. Teodora Donisan and then by expert faculty Dr. Eugene Yang.

Dr. Yang is professor of medicine of the University of Washington where he is medical director of the Eastside Specialty Center and the co-Director of the Cardiovascular Wellness and Prevention Program. Dr. Yang is former Governor of the ACC Washington Chapter and current chair of the ACC Prevention of CVD Section.

The following question refers to Section 3.4 of the 2021 ESC CV Prevention Guidelines. The question is asked by student Dr. Adriana Mares, answered first by early career preventive cardiologist Dr. Dipika Gopal, and then by expert faculty Dr. Michael Wesley Milks.

Dr. Milks is a staff cardiologist and assistant professor of clinical medicine at the Ohio State University Wexner Medical Center where he serves as the Director of Cardiac Rehabilitation and an associate program director of the cardiovascular fellowship. He specializes in preventive cardiology and is a member of the American College of Cardiology’s Cardiovascular Disease Prevention Leadership Council.

The following question refers to Section 4.8 of the 2021 ESC CV Prevention Guidelines. The question is asked by CardioNerds Academy Intern student Dr. Christian Faaborg-Andersen, answered first by UCSF resident Dr. Jessie Holtzman, and then by expert faculty Dr. Melissa Tracy.

Dr. Tracy is a preventive cardiologist, echocardiographer, Director of Cardiac Rehabilitation, and solid organ transplant cardiologist at Rush University.

The following question refers to Section 4.10 of the 2021 ESC CV Prevention Guidelines. The question is asked by CardioNerds Academy Intern student Dr. Christian Faaborg-Andersen, answered first by UCSD fellow Dr. Patrick Azcarate, and then by expert faculty Dr. Laurence Sperling.

Dr. Larry Sperling is the Katz Professor in Preventive Cardiology at the Emory University School of Medicine and Founder of Preventive Cardiology at the Emory Clinic. Dr. Sperling was a member of the writing group for the 2018 Cholesterol Guidelines, serves as Co-Chair for the ACC’s Cardiometabolic and Diabetes working group, and is Co-Chair of the WHF Roadmap for Cardiovascular Prevention in Diabetes.

The following question refers to Section 4.7 and figure 16 of the 2021 ESC CV Prevention Guidelines. The question is asked by CardioNerds Academy Intern Student Dr. Shivani Reddy, answered first by Fellow at Johns Hopkins Dr. Rick Ferraro, and then by expert faculty Dr. Roger Blumenthal.

Dr. Roger Blumenthal is professor of medicine at Johns Hopkins where he is Director of the Ciccarone Center for the Prevention of Cardiovascular Disease. He was instrumental in developing the 2018 ACC/AHA CV Prevention Guidelines. Dr. Blumenthal has also been an incredible mentor to CardioNerds from our earliest days.

The following question refers to Section 4.3 of the 2021 ESC CV Prevention Guidelines. The question is asked by CardioNerds Academy Intern Dr. Maryam Barkhordarian, answered first by medicine resident CardioNerds Academy House Chief Dr. Ahmed Ghoneem, and then by expert faculty Dr. Kim Williams.

Dr. Williams is Chief of the Division of Cardiology and is Professor of Medicine and Cardiology at Rush University Medical Center. He has served as President of ASNC, Chairman of the Board of the Association of Black Cardiologists (ABC, 2008-2010), and President of the American College of Cardiology (ACC, 2015-2016).

The following question refers to Section 3.3 of the 2021 ESC CV Prevention Guidelines. The question is asked by CardioNerds Academy Intern student Dr. Adriana Mares, answered first by Brigham & Women’s medicine intern & Director of CardioNerds Internship Dr. Gurleen Kaur, and then by expert faculty Dr. Allison Bailey.

Dr. Bailey is an advanced heart failure and transplant cardiologist at Centennial Heart. She is the editor-in-chief of the American College of Cardiology’s Extended Learning (ACCEL) editorial board and was a member of the writing group for the 2018 American Lipid Guidelines.

This question refers to Sections 3.2 and 3.3 of the 2021 ESC CV Prevention Guidelines. The question is asked by CardioNerds Academy Intern student Dr. Hirsh Elhence, answered first by Ohio State University Cardiology Fellow Dr. Alli Bigeh, and then by expert faculty Dr. Eugene Yang.

Dr. Yang is professor of medicine of the University of Washington where he is medical director of the Eastside Specialty Center and the co-Director of the Cardiovascular Wellness and Prevention Program. Dr. Yang is former Governor of the ACC Washington Chapter and current chair of the ACC Prevention of CVD Section.

Question and Answers

A 48-year-old Pakistani woman with rheumatoid arthritis comes to your clinic asking how she can reduce her risk of ASCVD. Her mother died of an MI at age 45, her father is healthy at age 79. Her calculated 10-year risk based on SCORE2 is 3%. SBP is 120 mmHg, LDL is 120 mg/dL. What is the next best step? 

A. Order an echocardiogram 
B. Schedule a follow-up appointment in 1 year 
C. Discuss initiating a statin 
D. Repeat lipid panel in 3-5 years 

Listen to the podcast episode!

Answer: C. Discuss Initiating a statin 

The absolute benefit derived from risk factor modification depends on the absolute risk of CVD and the absolute improvements in each risk factor category. Risk factor treatment recommendations are based on categories of CVD risk (“low-to-moderate”, “high”, and “very high”). The cut-off risk levels for these categories are numerically different for various age groups to avoid undertreatment in the young and to avoid overtreatment in the elderly. As age is a major driver of CVD risk, but lifelong risk factor treatment benefit is higher in younger people, the risk thresholds for considering treatment are lower for younger people as per the ESC guidelines. Treatment decisions should be made with shared decision-making valuing patient preference.  

Option A is INCORRECT- there is a lack of convincing evidence that echocardiography improves CVD risk reclassification, and it is NOT recommended to improve CV risk prediction. (Class III, LOE B) 

Option B is INCORRECT- simply doing nothing is not appropriate for this patient with elevated CVD risk.  

Option C is CORRECT- This patient has a seemingly low 10-year CVD risk based on SCORE 2 of 3% and her SBP is controlled; however, given her age she is considered as having high CVD risk, therefore treatment should be considered. Stepwise approach involves targeting LDL <100 (class IIa) so initiating a statin would be appropriate. This patient also carries several risk enhancing modifiers including Pakistani ethnicity, family history of premature CVD, and inflammatory comorbidity. All patients should be counseled on smoking cessation, lifestyle modifications, and target SBP <160 mmHg. 

Option D is INCORRECT- repeating a lipid panel without risk factor modification will not change treatment recommendations for this patient with elevated CVD risk.  

Main Takeaway 

In summary, when a patient <50 years old without established ASCVD has an estimated 10-year risk 2.5 to <7.5% they are considered high CVD risk and risk factor treatment should be considered. Risk modifiers should also be taken into consideration. 

*Of note- ACC/AHA guidelines recommend the ASCVD risk calculator to estimate 10-year risk and do not restructure CVD risk groups according to age groups. High risk in the ACC/AHA guidelines is considered to be >20%. 

Guideline Location

  • Table 5 and Figure 5, Page 3251
  • 3.2.3.4, Page 3253
  • 3.2.3, Figure 6 page 3252
  • 3.3, Pages 3258-3259 

Mr. Early M. Eye is a 55-year-old man with a history of GERD who is seeing you in clinic as he is concerned about his family history of early myocardial infarction and would like to discuss if he should be taking a statin for cardiovascular prevention. He has never smoked tobacco. His 10-year CVD risk is estimated to be 8%. Which imaging modality is recommended by the ESC guidelines to reclassify his CVD risk?

A. Coronary Artery Calcium (CAC) scoring
B. Echocardiography
C. Ankle brachial index
D. Contrast enhanced computed tomography coronary angiography (CCTA)
E. None of the above

Listen to the podcast episode!

The correct answer is A.

Coronary artery calcium (CAC) scoring can reclassify CVD risk upwards and downwards and should specifically be considered in patients with calculated risk scores that are around decision thresholds. CAC scores which are high-than-expected for age and sex increase estimated future CVD risk. Notably, CAC scoring may also be used to “de-risk” if CAC is absent or lower-than-expected. The 2021 ESC Prevention Guidelines give a Class IIb (LOE B) recommendation to consider CAC scoring to improve risk classification around treatment decision thresholds. However, one limitation of CAC is that it does not provide direct information on total plaque burden or stenosis severity. In addition, there is also a Class IIb (LOE B) recommendation to use plaque detection by carotid ultrasound as an alternative when CAC scoring is unavailable or not feasible. Plaque assessed through carotid ultrasound is defined as presence of wall thickening that is >50% greater than the surrounding vessel wall or a focal region with intima-media thickness measurement >1.5mm that protrudes into the lumen.

Similar to the ESC Prevention Guidelines, the 2019 ACC/AHA guidelines on primary prevention of CVD also have a Class IIa recommendation for using CAC score, and explicitly mention its use for adults at intermediate risk (>7.5% to <20% 10-year ASCVD risk) with cut-offs including >100 Agatson units to reclassify risk upwards and CAC of 0 to reclassify risk downwards. However, the guidelines also mention that clinicians should not down-classify risk in patients who have CAC of 0 if they are current smokers, have diabetes, have a family history of ASCVD, or have chronic inflammatory conditions. Furthermore, the 2018 ACC/AHA Cholesterol guidelines have a Class IIa recommendation that if CAC is 0, it is reasonable to withhold statin therapy and reassess risk in 5 to 10 years, as long as higher risk conditions that we just discussed are absent. If CAC is 1-99, it is reasonable to initiate statin therapy for patients ≥ 55 years of age.

Choice B is incorrect. Echocardiography is not recommended to improve CV risk prediction due to lack of convincing evidence that it improves CVD risk reclassification.

Choice C is incorrect. While the 2013 ESC guidelines mentioned that ABI may be considered as a risk modifier in CVD risk estimation, the newer 2021 guidelines state that ankle brachial index has limited potential in terms of reclassification risk, though an individual patient data meta-analysis showed there may be utility for women at intermediate risk. 12-27% of middle-aged individuals can have an abnormal ankle brachial index, defined as less than 0.9, of which 50-89% may not have typical claudication symptoms. Conversely, the 2019 ACC/AHA guidelines include ABI <0.9 as a risk-enhancing factor.

Choice D is incorrect. Coronary computed tomographic angiography (CCTA) has been shown in studies such as SCOT-HEART to have utility in predicting cardiac events in patients with stable chest pain and can identify coronary stenosis. It is not currently recommended by ESC guidelines for prognostic value or risk classification in asymptomatic patients.

According to the ESC guidelines, routine vascular testing or imaging other than CAC scoring or carotid ultrasound for plaque determination are not recommended (Class III, LOE B).

In terms of this patient’s family history of premature CVD, the ESC guidelines describe that even though family history is significantly associated with CVD in studies, it only marginally improves the prediction of CVD risk beyond conventional ASCVD risk factors. However, family history should still be obtained regularly when seeing patients, and if there is a positive family history of ASCVD, a comprehensive CVD risk assessment is warranted. Importantly, family history is not binary and those with a greater “dose” of family history (more relatives affected at earlier ages) may be at greater risk.

Main Takeaway

When a patient without established ASCVD has an estimated 10-year risk around treatment decision thresholds, CAC scoring is the best-established imaging modality to improve CVD risk stratification.

Guideline Location

Section 3.3.3, page 3259

Mrs. B is a 56-year-old African American woman with a past medical history significant for type 2 diabetes (HbA1C 7.6) and hypercholesterolemia. Her calculated ASCVD risk score today is 12.5% and her BMI is 24kg/m2. She is concerned about her high cholesterol levels despite being on a statin and feels that her diet is “not healthy enough.” She is interested in making dietary changes to help reduce her ASCVD risk. Which of the following recommendations is appropriate?

A. Sodium restriction to <3g /day will be of no benefit because she is not hypertensive.
B. Isocaloric substitution of saturated fat with polyunsaturated fat is associated with reduction of CHD risk.
C. Dietary fiber intake is associated with GI benefits but has no CV risk reduction benefit.
D. Supplementing diet with vitamins A, B, C and E helps reduce ASCVD risk.

Listen to the podcast episode! 

The correct answer is B.

Risk of CHD is reduced when dietary saturated fats are replaced with other foods having similar caloric values. The greatest reduction was observed when saturated fats were isocalorically replaced with polyunsaturated fats (↓25%), followed by monounsaturated fats (↓15%) and carbohydrates from whole grains (↓9%). This is a class 1a recommendation in the ESC guidelines and a class IIa recommendation in the 2019 ACC/AHA guidelines. Conversely, increased trans fatty acid intake is associated with increased CHD risk. A regulation of the European Union (EU) Commission has set the upper limit of trans fats to 2 g per 100 g of fat. The ACC/AHA guidelines recommend that the intake of trans fats should be avoided (a class III: harm).

Choice A is incorrect because dietary sodium restriction is recommended not only for control of blood pressure, but also for reduction of ASCVD risk. In a meta-analysis, salt reduction of 2.5 g/day resulted in a 20% reduction of ASCVD events (RR 0.80). Reduction of salt intake is a class 1 recommendation in the ESC guidelines compared to a class IIa recommendation in the 2019 ACC/AHA guidelines.

Choice C is incorrect because a 10 g/day higher fiber intake was associated with a 16% lower risk of stroke (RR 0.84) and a 6% lower risk of type 2 DM (RR 0.94). A high fiber intake may reduce postprandial glucose responses after carbohydrate-rich meals and also lower triglyceride levels. The Mediterranean diet is rich in fiber (it includes high intakes of fruits, vegetables, pulses and wholegrain products) and is a class I recommendation.

Choice D is incorrect because while vitamin supplementation has been associated with reduction in ASCVD risk in observational studies, intervention trials have failed to show any benefit. 

Main Takeaway:

A healthy diet is recommended as a cornerstone of CVD prevention in all individuals, independent of their underlying co-morbidities. Replacing saturated with unsaturated fats, reducing salt intake, and choosing a more plant-based diet that is rich in fiber can lower risk of CVD.

Guideline Location
Section 4.3.2, Page 3270

Ms. K.M. is a 40-year-old woman presenting to the outpatient clinic for a routine physical exam required for her employment as an airline stewardess. She states she has been in her usual good health but does experience occasional headaches and lightheadedness while in flight. On exam her BP was noted to be 170/90. The diagnosis of hypertension is confirmed during a subsequent clinic visit. What would be the most appropriate initial therapy recommendation(s) for Ms. K.M.?

A. Initiate single drug therapy with a beta-blocker.
B. Discuss and initiate lifestyle interventions
C. Initiate two-drug combination therapy with a thiazide-like diuretic, BB, CCB, or an ARB.
D. Both B and C 

Listen to the podcast episode! 

The correct answer is D.

The correct answer is D. Both B (lifestyle interventions) and C (initial combination therapy) are appropriate at this time.

Lifestyle interventions are indicated for all patients with high-normal BP or hypertension because they can delay the need for drug treatment or complement the BP-lowering effect of drug treatment (Class 1). Moreover, most lifestyle interventions have health benefits beyond their effect on BP.

Single-drug therapy will rarely achieve optimal BP control. Therefore, initial antihypertensive therapy with a combination of two drugs, preferably as a single-pill combination, is recommended for the management of HTN (Class 1). The only exceptions would be patients with a baseline BP close to the recommended target, who might achieve that target with a single drug, or very old (>80 years) or frail patients who may better tolerate a gentler reduction of BP.

Five major classes of BP-lowering drug therapy have shown benefit in reducing CV events; angiotensin-converting enzyme (ACE) inhibitors, angiotensin receptor blockers (ARBs), beta-blockers, calcium channel blockers (CCBs), and thiazide or thiazide-like diuretics.

A combination of an ACE inhibitor or ARB with a CCB or thiazide/thiazide-like diuretic is the preferred initial therapy for most patients with hypertension (Class 1). For those in whom treatment requires escalation to three drugs, a combination of an ACE inhibitor or ARB with a CCB and a thiazide/thiazide-like diuretic should be used (Class 1).

Resistant hypertension is defined as BP being uncontrolled despite treatment with optimal or best-tolerated doses of three or more drugs including a diuretic, and confirmed by ABPM or HBPM. Spironolactone is the most effective drug for lowering BP in resistant hypertension when added to existing treatment; however, the risk of hyperkalaemia is increased in patients with CKD. When spironolactone is not tolerated, amiloride, alpha-blockers, beta-blockers, or centrally acting drugs, such as clonidine, have evidence supporting their use. Renal denervation and device-based therapy may be considered for specific cases.

Beta-blockers should be used when there is a specific indication (e.g. angina, post myocardial infarction, arrythmia, HFrEF, or as an alternative to an ACE inhibitor or ARB in women of child-bearing potential). Combinations of an ACE inhibitor and an ARB are not recommended because of no added benefit on outcomes and increased risk of harm (Class III).

The diagnosis and treatment of hypertension in women is similar to that in men, except for women of child-bearing potential or during pregnancy, because of potential adverse effects of some drugs on the fetus, especially in the first trimester. In addition, the effect of oral contraceptive pills on the risk of developing or worsening hypertension should be considered.

Main Takeaway:

Combination drug therapies are more effective in lowering BP than monotherapy. BP management in women is similar to men with the exception of child-bearing potential given potential teratogenicity of some agents.

Guideline Location:

4.7.5.1-4, Page 3288

Figure 16, pg. 3287

The European Society of Cardiology Prevention guidelines currently recommend that low-dose colchicine (0.5mg/day) may be considered for the primary prevention of cardiovascular disease.

A. True
B. False

Listen to the podcast episode!

The correct answer is False. 

The European Society of Cardiology recommends that low-dose colchicine may be considered as an adjunctive therapy for secondary rather than primary prevention of cardiovascular disease in individuals whose risk factors are otherwise insufficiently controlled (Class IIb, LOE A). A broad evidence base currently supports that inflammation has pro-atherosclerotic effects and that reducing inflammation may reduce atherogenesis in high-risk patients.

The initial LoDoCo trial in 2013 first demonstrated a 10.7% absolute risk reduction in acute coronary syndrome, out of hospital cardiac arrest, and non-cardioembolic ischemic stroke with daily low-dose colchicine; however, results were clouded by small sample size. Subsequently, the CANTOS trial in 2017 demonstrated a 15% relative reduction in non-fatal myocardial infarction, non-fatal stroke, and cardiovascular death with Canakinumab, an anti-inflammatory monoclonal antibody inhibitor of interleukin-1. More recently, the COLCOT trial in 2019 studying patients with recent AMI and LoDoCo2 trial in 2021 studying patients with stable chronic CAD both demonstrated reductions in myocardial infarction, cardiovascular mortality, CVA, and ischemia-driven revascularization with colchicine 0.5mg/day. In the LoDoCo2 trial, stable CAD was defined either angiographically, by coronary CT, CAC >400, or history of CABG >10 years prior with evidence of failed grafts or angioplasty since that time.

In high-risk individuals with stable ischemic heart disease, the most recent evidence suggests that once daily low dose colchicine may reduce myocardial infarction and other ischemic events. Future studies may assess the biochemical markers including the trend of lipids and inflammatory markers to identify subpopulations that may benefit most from this therapy.

Main Takeaway:

Based upon the 2021 ESC Prevention Guidelines, clinicians may consider initiating low-dose colchicine (0.5mg/day) for secondary prevention of cardiovascular disease, particularly if other risk factors are insufficiently controlled or if recurrent CVD events occur despite optimal therapy.

Guideline Location: 

Section 4.10, page 3291.

A 62-year-old man with a history of non-obstructive coronary artery disease, heart failure with reduced ejection fraction (EF 30-35%), stage III chronic kidney disease, and type II diabetes mellitus presents to your clinic to establish care. His only medications are aspirin 81 mg daily and metformin 1000 mg BID, which he has taken since being diagnosed with diabetes mellitus 5 years ago. His hemoglobin A1c is 6.8%. What changes would you recommend to his medications at this time?

A. Start glipizide
B. Start saxagliptin
C. Start empagliflozin
D. No changes

Listen to the podcast episode!

The correct answer is C – start empagliflozin.

The Trials involving SGLT-2 inhibitors and GLP-1R agonists have shown cardiovascular benefits independent of glycemic control and metformin use.

The ADA recommends metformin as a first-line therapy for all patients with type 2 DM. The ESC also recommends metformin as first-line therapy but only in patients without ASCVD, CKD, or HF (Class I, LOE B). If a patient has ASCVD, metformin can be considered (Class IIa, LOE B). Rather, for those patients with type 2 DM and ASCVD, the ESC recommends the use of GLP-1R agonist or SGLT-2 inhibitors with proven outcome benefits to reduce CV and/or cardiorenal outcomes (Class I, LOE A). Additionally, for those with type 2 DM and either CKD or HFrEF, the ESC recommends the use of SGLT-2 inhibitor to improve outcomes (Class I, LOE A).

In contrast to the ADA, the view of the ESC is that metformin should be considered but is not mandatory first-line treatment in patients with diabetes and ASCVD or evidence of target organ damage. The initiation of metformin in such patients should not forego or delay the initiation of evidence-based SGLT2 inhibitors or GLP-1RAs.


Therefore, the next best step for our patient is to start an SGLT-2 inhibitor given his history of CAD, HF, and CKD. While this patient’s A1c goal is within the range recommended for patients with Type 2 DM and ASCVD (<7%), given his CAD, HF, and CKD an SGLT-2 inhibitor should still be added.

Saxagliptin is a DPP-4 inhibitor, a class of drugs that showed no effect of MACE but increased risk of HF hospitalization in patients with DM and existing.

Lifestyle management is a top priority for ASCVD prevention and management of DM. Lifestyle intervention lowers future microvascular and macrovascular risks as well as mortality in the longer term. Intensive lifestyle changes with low-calorie diets and mean weight losses in the region of 10 kg leads to remission of type 2 DM in around 46% of cases at 1 year and 36% by 2 years. Smoking cessation, a diet low in saturated fat and high in fiber, aerobic physical activity, strength training, and reduction in energy intake for weight optimization are all recommended for patient with diabetes mellitus (Class I).

Main Takeaway

In patients with Type 2 DM and ASCVD or end organ dysfunction, SGLT-2 inhibitors or GLP-1R agonists should be recommended regardless of background therapy or glycemic control. For patients with type 2 diabetes mellitus and CKD or HFrEF, SGLT-2 inhibitor is recommended.

Guideline Location

Section 4.8.1, Pages 3289-90.

While you are on holiday break visiting your family, your aunt pulls you aside during the family gathering to ask a few questions about your 70-year-old uncle. He has hypertension, hyperlipidemia, type 2 diabetes mellitus, and moderate chronic obstructive pulmonary disease. His medications include Fluticasone/Salmeterol, Tiotropium, Albuterol, Lisinopril, Simvastatin, and Metformin. She is very concerned about his risk for heart disease as he has never had his “heart checked out.” She asks if the presence of COPD increases his chance of having heart disease. Which of the following statements would best answer her question?

A. Systemic inflammation and oxidative stress caused by COPD promote vascular remodeling and a paradoxical ‘anticoagulant’ state affecting all vasculature types.
B. Although chronic COPD is associated with increased cardiovascular events, individual exacerbations have no impact on risk of cardiovascular events.
C. Patients with mild-moderate COPD are 8-10x more likely to die from atherosclerotic cardiovascular disease than respiratory failure.
D. Cardiovascular mortality increases proportionally with an increase in forced expiratory volume in 1 second (FEV1)

Listen to the podcast episode!

The correct answer is C.

Patients with mild-moderate COPD are 8-10x more likely to die from atherosclerotic cardiovascular disease than respiratory failure. Patients with COPD have a 2-3-fold increased risk of CV events compared to age-matched controls even when adjusted for tobacco smoking, a shared risk factor. This can be partly explained by other common risk factors including aging, hypertension, hyperlipidemia, and low physical activity.

Interestingly, CVD mortality increases proportionally with a decrease (rather than increase) in FEV1, making answer choice D wrong (28% increase CVD mortality for every 10% decrease in FEV1). Additionally, COPD exacerbations and related infections are associated with a 4x increase in CVD events, making answer choice B incorrect.

COPD has several effects on the vasculature which creates a ‘procoagulant’ not ‘anticoagulant’ effect on all vascular beds. This is associated with increased risk of cognitive impairment due to cerebral microvascular damage as well as increased risk of ischemic and hemorrhagic stroke.

Main Takeaway

The presence of COPD (even mild to moderate) has a significant impact on the incidence of non-fatal coronary events, stroke, and cardiovascular mortality mediated by inherent disease process and progression, risk factors (smoking, aging, hypertension, and hyperlipidemia), and systemic inflammation altering vasculature creating a ‘procoagulant’ effect. The ESC gives a Class I indication (LOE C) to investigate for ASCVD and ASCVD risk factors in patients with COPD.

Guideline Location

3.4.5, Page 3264.

Please read the following patient vignettes and choose the FALSE statement.

A. A 39-year-old man who comes for a regular physical, has normal vitals and weight, denies any significant past medical or family history – does not need systematic cardiovascular disease (CVD) assessment.
B. A 39-year-old woman who comes for a regular physical, has normal vitals and weight, and has a history of radical hysterectomy (no other significant past medical or family history) – could benefit from systematic or opportunistic CVD assessment.
C. A 39-year-old woman who comes for a regular physical, has normal vitals except for a BMI of 27 kg/m2 and a family history of hypertension – requires a systematic global CVD assessment.
D. A 39-year-old man who comes for a regular physical, has normal vitals and weight, and has a personal history of type I diabetes – requires a systematic global CVD assessment.

Listen to the podcast episode! 

The correct answer is C.

Option A is an accurate statement, as systematic CVD risk assessment is not recommended in men < 40 years-old and women < 50 years-old, if they have no known cardiovascular (CV) risk factors. (Class III, level C)

Option B is an accurate statement, as this patient had a radical hysterectomy, which means the ovaries have been removed as well and she is considered postmenopausal. Systematic or opportunistic CV risk assessment can be considered in men > 40 years-old and women > 50 years-old or postmenopausal, even in the absence of known ASCVD risk factors. (Class IIb, level C)

Option C is a false statement and thus the correct answer, as the recommendations for global screening in this patient are not as strong and would require shared decision making. Opportunistic screening of blood pressure can be considered in her, as she is at risk for developing hypertension. Blood pressure screening should be considered in adults at risk for the development of hypertension, such as those who are overweight or with a known family history of hypertension. (Class IIa, level B)

Option D is an accurate statement, as systematic global CVD risk assessment is recommended in individuals with any major vascular risk factor (i.e., family history of premature CVD, familial hyperlipidemia, CVD risk factors such as smoking, arterial hypertension, DM, raised lipid level, obesity, or comorbidities increasing CVD risk). (Class I, level C)

Additional learning points:

Do you know the difference between opportunistic and systematic CVD screening?

  • Opportunistic screening refers to screening without a predefined strategy when the patient presents for different reasons. This is an effective and recommended way to screen for ASCVD risk factors, although it is unclear if it leads to benefits in clinical outcomes.
  • Systematic screening can be done following a clear strategy formally evaluating either the general population or targeted subpopulations (i.e., type 2 diabetics or patients with significant family history of CVD). Systematic screening results in improvements in risk factors but has no proven effect on CVD outcomes.

Main Takeaway

Systematic CVD risk assessment in the general population without CV risk factors does not seem to be cost effective and has unclear benefits on outcomes, although it does lead to increased detection of potentially actionable CV risk factors. Risk assessment is not a one-time event and should be repeated (e.g., every 5 years), but there is no clear data to guide intervals.

Guideline Location

Section 3.1, page 3236; Table on page 3242.

Mr. A is a 28-year-old man who works as an accountant in what he describes as a “desk job” setting. He shares that life got “a little off-track” for him in 2020 between the COVID-19 pandemic and a knee injury. His 2022 New Years’ resolution is to improve his overall cardiovascular and physical health. He has hypertension and a family history of premature ASCVD in his father, who died of a heart attack at age 50. Prior to his knee injury, he went to the gym 3 days a week for 1 hour at a time, split between running on the treadmill and weightlifting. He has not returned to the gym since his injury and has been largely sedentary, although he is trying to incorporate a 20-minute daily walk into his routine. Which of the following exercise-related recommendations is most appropriate?

A. A target of 75-150 minutes of vigorous-intensity or 150-300 minutes of moderate-intensity aerobic physical exercise weekly is recommended to reduce all-cause mortality, CV mortality, and morbidity.
B. Bouts of exercise less than 30 minutes are not associated with favorable health outcomes.
C. Exercise efforts should be focused on aerobic activity, since only this type of activity is associated with mortality and morbidity benefits.
D. Light-intensity aerobic activity like walking is expected to have limited health benefits for persons with predominantly sedentary behavior at baseline.

Listen to the podcast episode! 

The correct answer is A.

There is an inverse relationship between moderate-to-vigorous physical activity and CV morbidity/mortality, all-cause mortality, and incidence of type 2 diabetes, with additional benefits accrued for exercise beyond the minimum suggested levels. The recommendation to “strive for at least 150-300 min/week of moderate-intensity, or 75-150 min/week of vigorous-intensity aerobic physical activity, or an equivalent combination thereof” is a Class 1 recommendation per the 2021 ESC guidelines, and a very similar recommendation (at least 75 minutes of vigorous-intensity or 150 minutes of moderate-intensity activity) is also Class 1 recommendation per 2019 ACC/AHA primary prevention guidelines. Both the ESC and ACC/AHA provide examples of activities grouped by absolute intensity (the amount of energy expended per minute of activity), but the ESC guidelines also offer suggestions for measuring the relative intensity of an activity (maximum/peak associated effort) in Table 7, which allows for a more individualized, customizable approach to setting activity goals. Importantly, individuals who are unable to meet minimum weekly activity recommendations should still be encouraged to stay as active as their abilities and health conditions allow to optimize cardiovascular and overall health.

Choice B is incorrect, as data suggests physical activity episodes of any duration, including <10 min, are associated with favorable outcomes like all-cause mortality benefit. The duration of a single exercise bout is less correlated with health benefits than the total physical activity time accumulated per week.

Choice C is incorrect. Per the ESC guidelines, it is a class 1 recommendation to perform resistance exercise, in addition to aerobic activity, on 2 or more days per week to reduce all-cause mortality. Data indicate that the addition of resistance exercise to aerobic activity is associated with lower risks of total CV events and all-cause mortality, so it’s expected that a combination of weightlifting and aerobic activity may be more beneficial for than either type of activity alone. The 2019 ACC/AHA prevention guidelines do not make a formal recommendation regarding resistance exercise; they do note that it has multiple health benefits (e.g., BP-lowering, improved glycemic control) though state its association with ASCVD risk reduction is unclear. 

Choice D is incorrect: sedentary time is independently associated with greater risk for several major chronic diseases and mortality. Reducing sedentary time for inactive adults and adding in light-intensity physical activity (as little as 15 minutes daily) is a class 1 recommendation to reduce all-cause and CV mortality and morbidity. The 2019 ACC/AHA guidelines suggest that reduced sedentary behavior may be “reasonable for ASCVD risk reduction” (Class 2b). Assuming our patient has had predominantly sedentary behavior, starting with a 20-minute daily walk can provide initial health benefits while working up to more and higher-intensity activity.

Main Takeaway

Physical activity should be individually assessed and prescribed in terms of frequency, intensity, time (duration), type, and progression.

Guideline Location

Section 4.3.1, Pages 3268-3269, Table 7

Ms. DW is a 67-year-old woman with a history of coronary artery disease and prior percutaneous coronary intervention in 2019 with a drug-eluting stent to the proximal left anterior descending artery. They have transitioned to your clinic from a previous provider, and their LDL is 134 mg/dL. What would be the ESC recommended goal LDL-C level for this patient? 

A. <30 mg/dL
B. <55 mg/dL
C. <70 mg/dL
D. <100 mg/dL

Listen to the podcast episode!

The correct answer is B.

The ESC guidelines outline a robust LDL-C reduction goal of <55mg/dL (<1.4mmol/L) and ≥50% reduction from baseline in those with known atherosclerotic cardiovascular disease, with the highest possible Class I (LOE A) recommendation for this goal. A goal LDL-C <55mg/dL with ≥50% reduction from baseline should also be considered in apparently healthy persons <70 years of age who are at very high risk (Class IIa, LOE C). 

To achieve these goals, the guidelines recommend a stepwise approach to treatment including dietary, lifestyle, and medical management. Recognizing that lower LDL-C is better, the guidelines recommend liberal intensification of treatment especially if using submaximal doses of generic or low-cost statins and side effects are not apparent. High-intensity statin is recommended to be prescribed to the highest tolerated dose to reach LDL-C goals set for each specific risk group (Class I, LOE A). If these goals are not achieved with the maximum tolerated dose of a statin, combination therapy with ezetimibe is recommended (Class I, LOE B).

Choice A is incorrect. An LDL-C <30mg/dL is a more significant reduction than that recommended by the guidelines, even for patients with known ASCVD. Notably, for patients with ASCVD who experience a second vascular event within 2 years while taking maximum tolerated statin-based therapy, an LDL-C goal of <1.0 mmol/L (40 mg/dL) may be considered.

Choice C is incorrect. The ESC prevention guidelines recommend considering a goal of <70mg/dL for patients in the primary prevention setting at high risk who are <70 years of age (Class IIa, LOE C). (Recall again that for those at very high risk the primary prevention recommendation is target LDL-C <55 mg/dL).

Choice D is incorrect. LDL-C <100mg/dL was a frequently cited goal in older iterations of various prevention and lipid guidelines. As the data has shifted to support lower LDL-C goals, this is not a noted goal within the 2021 ESC prevention guidelines for patients <70 years of age.

Main Takeaway

Lower is better when it comes to LDL-C

For those with known atherosclerotic cardiovascular disease, liberal intensification of lipid lowering treatment is recommended. 

Guideline Location

Section 4.6.2.1, page 3276-3279, Figure 6 on page 3252, Figure 7 on page 3253

A 70-year-old man with a history of hypertension, diabetes, hyperlipidemia, peptic ulcer disease with a prior upper GI bleed, as well as coronary artery disease presents to clinic. About one year ago he suffered an NSTEMI treated with percutaneous coronary intervention to the mid LAD. He is feeling well and able to walk 1 mile daily with no anginal symptoms. He is currently taking aspirin 81 mg daily, ticagrelor 90 mg BID, atorvastatin 40 mg daily, metoprolol 25 mg BID, lisinopril 5 mg daily, and lansoprazole 15mg daily. He has a preserved ejection fraction. His BP in clinic is 110/70 and HR is 65 bpm. His LDL is 50 mg/dL. What do you recommend for his further management?

A. Switch ticagrelor to clopidogrel, continue indefinitely
B. Stop ticagrelor, continue aspirin indefinitely
C. Continue aspirin + ticagrelor indefinitely
D. Stop ticagrelor, start rivaroxaban 2.5 mg BID

Listen to the podcast episode!

The correct answer is B – stop ticagrelor, continue aspirin indefinitely.

Twelve months of DAPT is recommended for acute coronary syndromes (Class I, LOE A). Long-term secondary prevention with dual anti-thrombotic therapy (DAPT > 12 months with a P2Y12 inhibitor and low-dose aspirin or low-dose rivaroxaban 2.5mg BID with low-dose aspirin) may be considered for patients who are at high ischemic risk without high risk of bleeding (Class IIa, LOE A). However, this patient is at increased bleeding risk (peptic ulcer disease with prior GI bleeding) and has no ischemic symptoms, and so neither would be recommended.

Main Takeaway

In summary, 12 months of DAPT is recommended after ACS. Prolonged DAPT or low-dose rivaroxaban may be considered with high ischemic risk and low bleeding risk.

Guideline Location

Section 6.1, Pages 3294-3295.

Medically supervised cardiac rehabilitation programs after ASCVD events and for patient with heart failure carries a Class I recommendation. However, placement of referrals, uptake and enrollment after referral, and rigor of rehabilitation all remain inconsistent. What minimum cumulative duration of cardiac rehabilitation has been chosen as a threshold of effectiveness for cardiac rehabilitation by the European Society of Cardiology?

A. 100-300 minutes, 10 sessions
B. 300-500 minutes, 16 sessions
C. 500-700 minutes, 22 sessions
D. 700-1000 minutes, 28 sessions
E. >1000 minutes, 36 sessions

Liston to the podcast episode!

The correct answer is E: >1000 minutes across 36 sessions.

Cardiac rehabilitation is a comprehensive, multidisciplinary intervention not just including exercise training and physical activity counselling, but also education, risk factor modification, diet/nutritional counselling, and vocational and psychosocial support.

A broad evidence base demonstrates that multidisciplinary cardiac rehabilitation and prevention programs after ASCVD events or revascularization reduce recurrent cardiovascular hospitalizations, myocardial infarction, and cardiovascular mortality. In patients with chronic HF (mainly HFrEF), exercise based cardiac rehabilitation (EBCR) may improve all-cause mortality, reduce hospital admissions, and improve exercise capacity and quality of life. Such programs include a wide array of activities including physical activity, risk factor modification, psychosocial support, nutrition counseling, and more. Despite the heterogenous design of clinical trials, cardiac rehabilitation has been shown to be a cost-effective intervention.

Based upon the available review data, the European Association of Preventive Cardiology and the European Society of Cardiology proposed minimum standards for secondary prevention cardiac rehabilitation programs. Based upon a comprehensive review of the literature, ESC recommends that cardiac rehabilitation be multidisciplinary, supervised by health professionals, and start as soon as possible after a cardiovascular event. Cardiac rehabilitation should include both aerobic and muscular resistance tailored to the fitness level of the participant, should carry a duration of >1000 minutes in total, and should exceed 36 sessions total.

While uptake remains limited, electronic prompts within the medical record and automatic referrals should be considered to enhance referral and participation. Future research should continue to explore the benefit of home-based cardiac rehabilitation with or without telemonitoring. Lastly, studies have shown that uptake remains lower among women, and targeted programs should be undertaken to address such disparities.

Main Takeaway

Current European Society of Cardiology guidelines provide a Class I (LOE A) recommendation for the participation in multidisciplinary cardiac rehabilitation programs for the secondary prevention of ASCVD events including revascularization and in individuals with heart failure (mainly HFrEF) to improve patient outcomes.

Guideline Location

Section 4.11, Page 3292.

You are seeing a 45-year-old woman with a past medical history of hypertension, overweight status, hyperlipidemia, and active tobacco use disorder. Her BMI is 27 kg/m2, BP is 150/75, HbA1C is 5.8%, total cholesterol is 234 mg/dL, HDL is 59 mg/dL, and LDL is 155 mg/dL. She is from Romania, a country with very high CVD risk. Which of the following statements is CORRECT?

A. LDL-C needs to be decreased by at least 50%, as small absolute LDL-C reductions would not provide clinical benefit

B. Hypertension is not an important CVD risk factor in our patient, as she is young.

C. Prediabetes is not a significant CV risk factor for our patient, as she is not yet diabetic.

D. Smoking confers a higher CVD risk for women than for men.

E. Her weight does not increase her CVD risk, as she is overweight rather than obese.

Listen to the podcast episode!

The correct answer is D – Smoking confers a higher CVD risk for women than for men.

Prolonged smoking increases the CVD risk more in women than in men. Our patient is 45 years old. CVD risk in smokers < 50 years-old is 5x higher than in non-smokers. Of note, smoking is responsible for 50% of all avoidable deaths in smokers and a lifetime smoker will lose 10 years of life, on average. Secondhand smoke and smokeless tobacco can also increase the CVD risk.

Option A is incorrect. The SCORE2 risk chart for populations at very high CVD risk places her at a 14% (very high) 10-year risk for myocardial infarction, stroke, or cardiovascular death. She would derive benefit even from incremental reductions in LDL-C values. The absolute benefit of lowering LDL-C depends on both the absolute risk of ASCVD and the absolute reduction in LDL-C, so even a small absolute reduction in LDL-C may be beneficial in high- or very-high-risk patients. Furthermore, the reduction in CVD risk is proportional to the decrease in LDL-C, irrespective of the medications used to achieve such change. This remains true even when lowering LDL-C values to < 55 mg/dl.

Option B is incorrect. Hypertension is a major cause of CVD regardless of age, and the risk of death from either CAD or stroke increases linearly from BP levels as low as 90 mmHg systolic and 75 mmHg diastolic upwards. Particularly relevant for our patient, lifetime BP evolution differs in women compared to men, potentially resulting in an increased CVD risk at lower BP thresholds.

Option C is incorrect. Type 1 DM, type 2 DM, and prediabetes are all independent risk factors for ASCVD. Of note, it would be important to address this risk factor with our patient, as women who develop type 2 diabetes have a particularly high risk for stroke.

Option E is incorrect. All-cause mortality is lowest at a BMI on 20-25 kg/m2 in apparently healthy patients. Even overweight patients are at increased CVD risk. There is a linear relationship between BMI and mortality in non-smokers and a J-shaped relationship in ever-smokers. In patients with heart failure, a lower mortality risk has been observed with higher BMI – the “obesity paradox.” It would be important to evaluate the waist circumference in our patient, as both BMI and waist circumference are associated with ASCVD risk.

Main Takeaway

The main ASCVD risk factors are hyperlipidemia (elevated apolipoprotein-B-containing lipoproteins, of which LDL-C is most abundant), hypertension, cigarette smoking, diabetes, and adiposity. Identifying patients who will benefit most from ASCVD risk factor treatment is central to ASCVD prevention efforts. In general, the higher the absolute CVD risk, the higher the absolute benefit of risk factor treatment, and thus the lower the number needed to treat to prevent one CVD event during a period of time.

Guideline Location

Section 3.2.1., pages 3236, 3242, 3243.

Ms. Soya M. Alone is a 70-year-old woman of Bangladeshi ethnicity with a history of anxiety and depression. She currently lives at home by herself, does not have many friends and family that live nearby, and has had a tough year emotionally after the passing of her husband. She spends most of her time in bed with low daily physical activity and has experienced more weakness and exhaustion over the past year along with loss of muscle mass. Which of the following are potential risk modifiers in this patient when considering her risk for CVD?

A. Bangladeshi ethnicity
B. Psychosocial factors
C. Frailty
D. History of anxiety and depression
E. All of the above

Listen to this podcast episode! 

The correct answer is E – All of the above.

Traditional 10-year CVD risk scores do not perform adequately in all ethnicities. Therefore, multiplication of calculated risk by relative risk for specific ethnic subgroups should be considered (Class IIa, LOE B). Individuals from South Asia have higher CVD rates. The ESC guidelines recommend using a correction factor by multiplying the predicted risk by 1.3 for Indians and Bangladeshis, and 1.7 for Pakistanis. These correction factors are derived from data from QRISK3. In the UK, the QRISK calculator algorithm has been derived and validated in 2.3 million people to estimate CVD risk in different ethnic groups and unlike other calculators, it counts South Asian origins as an additional risk factor. The reasons for such differences remain inadequately studied, as do the risks associated with other ethnic backgrounds. Barriers to developing accurate risk prediction tools include the wide heterogeneity amongst the population.

The 2019 ACC/AHA guidelines also list high-risk race/ethnicities such as South Asian ancestry as a risk-enhancing factor. However, there is no separate pooled cohort equation for different ethnicities, and consideration should be given that the pooled cohort equations will underestimate ASCVD risk in South Asians.

Psychosocial stress including loneliness and critical life events are associated, in a dose-response pattern, with the development and progression of ASCVD, with relative risks between 1.2 and 2.0. Conversely, indicators of mental health, such as optimism and a strong sense of purpose, are associated with lower risk. While there is not a specific way proposed by the guidelines for psychosocial factors to improve risk classification, it is important to screen patients with ASCVD for psychological stress, and clinicians should attend to somatic and emotional causes of symptoms as well. The ESC guidelines give a Class IIa (LOE B) recommendation for assessment of stress symptoms and psychosocial stressors.

This patient should also be formally screened for frailty, which is not the same as aging but includes factors such as slowness, weakness, low physical activity, exhaustion and shrinking, and makes her more vulnerable to the effect of stressors and is a risk factor for both high CV and non-CV morbidity and mortality. However, the ability of frailty measures to improve CVD risk prediction has not been formally assessed, so the guidelines do not recommend integrating it into formal CVD risk assessment. Frailty may however, influence treatment as it can help build an individualized care plan.

Mental disorders such as anxiety and depression are associated with the development of CVD as well. Detrimental effects may be potentially caused by unhealthy lifestyle, increased exposure to socioeconomic stressors, and cardiometabolic side-effects of medications. The ESC guidelines give a Class 1 (LOE C) recommendation that mental disorders with either significant functional impairment or decreased use of healthcare systems be considered as influencing total CVD risk.

Main Takeaway

Psychosocial stress and frailty are associated with risk of ASCVD and should be assessed in patients when considering CVD risk. In addition, current risk scores may under-or over-estimate CVD risk in different ethnic minority groups.

Guideline Location

Section 3.3.1, 3.3.2, 3.3.4, 3.4.10, page 3258 – 3259, 3265 – 3266

Your patient mentions that she drinks “several” cups of coffee during the day. She also describes having a soda daily with lunch and occasionally a glass of wine with dinner. Which of the following recommendations is appropriate?

 A. Coffee consumption is not harmful and may even be beneficial, regardless of the number of drinks per day.
B. Drinking two glasses of wine/day is safe from a cardiovascular prevention standpoint.
C. Soft drinks (and other sugar-sweetened beverages) must be discouraged.
D. None of the above

Listen to this podcast episode! 

The correct answer is C.  Soft drinks (and other sugar-sweetened beverages) must be discouraged.

Sugar-sweetened beverages have been associated with a higher risk of CAD and all-cause mortality. The ESC guidelines give a class I recommendation for restriction of free sugar consumption (in particular sugar-sweetened beverages) to a maximum of 10% of energy intake. This is a class IIa recommendation in the ACC/AHA guidelines.

Choice A is incorrect because: the consumption of nine or more drinks a day of non-filtered coffee (such as boiled, Greek, and Turkish coffee and some espresso coffees) may be associated with an up to 25% increased risk of ASCVD mortality. Moderate coffee consumption (3-4 cups per day) is probably not harmful, and perhaps even moderately beneficial.

Choice B is incorrect: It is a class I recommendation to restrict alcohol consumption to a maximum of 100 g per week. The standard drink in the US contains 14 g of alcohol, so 100 mg of alcohol translate to:

o   84 ounces of beer (5% alcohol)

o   Or 56 – 63 ounces of malt liquor (75% alcohol) or

o   Or 35 ounces of wine (12% alcohol) or ONE 5 fl oz glass of wine/day.

o   Or 31.5 ounces of distilled spirits (40% alcohol).

The ACC/AHA guidelines recommended limiting alcohol consumption only for the management of hypertension to: ≤2 drinks daily for men and: ≤1 drink daily for women.

Main Takeaway

The main takeaway: ASCVD risk reduction can be achieved by restricting sugar-sweetened beverages to a maximum of 10% of energy intake.

Guideline Location

Section 4.3.2, Page 3271

True or False: For patients with established ASCVD, secondary prevention entails adding a PCSK9 inhibitor if goal LDL is not met on maximum tolerated doses of a statin and ezetimibe.

Listen to this podcast episode! 

The correct answer is True.

The ultimate on-treatment LDL-C goal of <55 mg/dL (<1.4 mmol/L) and a reduction of at least ≥50% from baseline should be considered for primary prevention of persons <70 years of age at very high risk (Class IIa) and in those with established ASCVD (Class I).

It is recommended that a high-intensity statin is prescribed up to the highest tolerated dose to reach these LDL-C goals (Class I).

The combination of statin with ezetimibe brings a benefit that is in line with meta-analyses showing that LDL-C reduction has benefits independent of the approach used.  The beneficial effect of ezetimibe is also supported by genetic studies. Together, these data support the position that ezetimibe should be considered as second-line therapy, either on top of statins when the therapeutic goal is not achieved (Class I), or when a statin cannot be prescribed (Class IIa).

PCSK9 inhibitors (monoclonal antibodies to PCSK9) decrease LDL-C by up to 60%, either as monotherapy or in addition to the maximum tolerated dose of statin and/or other lipid-lowering therapies, such as ezetimibe. Their efficacy appears to be largely independent of background therapy. Among patients in whom statins cannot be prescribed, PCSK9 inhibition reduced LDL-C levels when administered in combination with ezetimibe. Both alirocumab and evolocumab effectively lower LDL-C levels in patients who are at high or very high CVD risk, including those with DM, with a large reduction in future ASCVD events.

Therefore, for those who do not meet LDL-C goals with maximally tolerated doses of both a statin and ezetimibe, combination therapy including a PCSK9 inhibitor may be considered for primary prevention of patients at very high risk but without familial hypercholesterolemia (Class IIa) and is recommended for secondary prevention for those with established ASCVD (Class I). In addition, for very-high-risk FH patients (that is, with ASCVD or with another major risk factor) who do not achieve their goals on a maximum tolerated dose of a statin and ezetimibe, combination therapy including a PCSK9 inhibitor is recommended (Class I).

Main Takeaway

Statins, ezetimibe, and PCSK9 inhibitors should be used in a stepwise approach to achieve target lipid lowering goals in accordance with their risk profile.

Guideline Location

Page 3279, Sections 4.6.3.1.4, 4.6.3.1.5

Figure 13 page 3278; recommendation table page 3279.

A 74-year-old man with a history of hypertension, chronic kidney disease, and gastroesophageal reflux presents with chest pain and is found to have an NSTEMI due to an obstructive lesion in the proximal LAD. One drug-eluting stent is placed, and he is started on dual antiplatelet therapy with aspirin and clopidogrel. He is concerned about the risk of bleeding from his gastrointestinal tract. What would you recommend to reduce his risk of bleeding?

A. Lansoprazole, a proton pump inhibitor
B. Famotidine, a histamine-2 blocker
C. Calcium carbonate, an antacid
D. None, proton pump inhibitors are contraindicated.

Listen to this podcast episode! 

The correct answer is A.

The ESC recommends that patients at high risk for GI bleeding who are receiving antiplatelet therapy take proton pump inhibitors (Class I, LOE A). High risk for bleeding includes patients who are age ≥65, history of peptic ulcer disease, Helicobacter pylori infection, dyspepsia or GERD symptoms, chronic renal failure, diabetes mellitus, and concomitant use of other antiplatelet agents, anticoagulants, nonsteroidal anti-inflammatory drugs, or steroids.

Coadministration of proton pump inhibitors that specifically inhibit CYP2C19 (omeprazole or esomeprazole) may reduce the pharmacodynamic response to clopidogrel. Although this interaction has not been shown to affect the risk of ischemic events, coadministration of omeprazole or esomeprazole with clopidogrel is not recommended.

Main Takeaway

In patients with high gastrointestinal bleeding risk who are receiving antiplatelet therapy, proton pump inhibitors are recommended. Omeprazole and esomeprazole may reduce the efficacy of clopidogrel and should not be used concomitantly with clopidogrel.

Guideline Location

Section 4.9.3, Page 3291

Figure 13 page 3278; recommendation table page 3279.

A 60-year-old Black woman with a history of hypertension and heart failure with reduced ejection fraction (EF 40%) presents to clinic for follow-up. She is currently doing well with NYHA class II symptoms. She is taking carvedilol 25 mg BID, sacubitril/valsartan 97/103 mg BID, and spironolactone 25 mg daily, all of which have been well tolerated. In clinic, her BP is 125/80 mmHg, and her HR is 55 bpm. Routine labs are within normal limits including Cr of 1.0, K of 4.0, and HbA1c of 6.0. What is the most appropriate next step in her management?

A. No change in management
B. Reduce beta blocker
C. Add an SGLT2 inhibitor (dapagliflozin or empagliflozin)
D. Add vericiguat
E. Add hydralazine/isosorbide dinitrate

Listen to this podcast episode!

The correct answer is C – Add an SGLT2 inhibitor (dapagliflozin or empagliflozin)

For patients with symptomatic HFrEF, neurohormonal antagonists (ACEi, ARB, ARNI; BB; MRA) improve survival and reduce the risk of HF hospitalization. This patient is already on these agents. The addition of an SGLT2 inhibitor on top of neurohormonal blockade reduces the risk of CV death and worsening HF in patients with symptomatic HFrEF and is the next best step for this patient (Class I, LOE A).

Vericiguat may be considered in patients with symptomatic HFrEF with HF worsening despite already being on maximally tolerated neurohormonal blockade (Class IIb, LOE B), but first-line therapies should be started first.

Hydralazine/Isosorbide dinitrate should be considered in self-identified Black patients or people who have EF ≤ 35% or <45% with dilated LV with class III-IV symptoms despite maximally tolerated neurohormonal blockade (Class IIa, LOE B), but is not the next best step here.

She is tolerating the beta blocker without adverse effects so there is no reason to decrease the dosage.

Main Takeaway

In patients with symptomatic HFrEF (EF ≤ 40%), SGLT2 inhibitors are considered first line therapy in addition to ACE-I/ARB/ARNI, BB, and MRAs to reduce the risk of HF hospitalization and death. Importantly this is irrespective of presence of diabetes.

Guideline Location

Section 6.2, page 3295-3296

Figure 13 page 3278; recommendation table page 3279.

True or False: A 70-year-old male has an estimated 10-year ASCVD risk (using SCORE2-OP) of 7.5% which confers a very high CVD risk and necessitates treatment with a statin. 

TRUE 

FALSE 

Explanation  

FALSE – CVD risk thresholds for risk factor treatment are higher in apparently healthy people 70 years and older in order to prevent overtreatment in the elderly. A 10-year CVD risk 15% is considered “very high risk” for individuals 70 years of age (compared to a 7.5% cut-off for “very high risk” in younger patients <50 years old). For these patients, treatment of ASCVD risk factors, including lipid-lowering medications, is recommended (class IIb).  Lifetime benefit of treatment in terms of time gained free of CVD is lower in older people. The SCORE2-OP algorithm estimates 5-year and 10-year fatal and non-fatal CVD events adjusted for competing risks of non-CVD mortality. Treatment and risk stratification should (as with all patients) be individualized.  

For patient >70 years of age, a 10-year CVD risk of 7.5 to <15% is considered “high risk”, and treatment of risk factors should be considered taking CVD risk modifiers, frailty, lifetime treatment benefit, comorbidities, polypharmacy, and patient preference into account.  

For patient >70 years of age, a 10-year CVD risk of <7.5 is considered “low-to-moderate risk” and would generally not qualify for risk factor treatment unless one or several risk modifiers are present.  

Smoking cessation, lifestyle recommendations and a SBP <160 mmHg are recommended for all. 

Main Takeaway 

  • CVD risk assessment for patients 70-years and older is estimated using the SCORE2-OP algorithm. A predicted 10-year CVD risk score of 15% confers a very high CVD risk, however, this it is a class IIb indication to initiate/intensify lipid lowering therapies in these patients. Decision should be individualized and based on benefits vs risk assessment. 

Guideline Loc. 

  • 3.2.3.5 

Ms. Ruma Toid is a 65-year-old African American woman who presents to your clinic in Ohio for routine follow up. She has a history of rheumatoid arthritis, hypertension, obesity, and sleep apnea. Her medications include methotrexate and atenolol. Her blood pressure in the office is 120/80 mmHg, heart rate 68 bpm, and oxygen saturation 99% on room air. Recent lipid testing revealed total cholesterol 165 mg/dL, HDL 42 mg/dL, and LDL 118 mg/dL. She was recently advised to talk to her doctor about taking a statin due to her risk factors but in the past has heard negative things about those medications and would like your advice on next steps. Her calculated ASCVD risk score based on the Pooled Cohort Equation is 7%. Which of the following choices would be the next step?

A

She is at borderline risk for ASCVD events. A statin is not indicated at this time.

B

Due to her history of rheumatoid arthritis, her calculated ASCVD risk should be multiplied by 1.5, yielding an ASCVD risk of 10.5% placing her in the intermediate risk category. Moderate intensity statin would be indicated.

C

When other risk factors are present, rheumatoid arthritis is no longer an enhancing risk factor.

D

Statins are contraindicated when taking methotrexate.

Explanation

The correct answer is B. Due to her history of rheumatoid arthritis, her calculated ASCVD risk should be multiplied by 1.5, yielding an ASCVD risk of 10.5% placing her in the intermediate risk category. Moderate intensity statin would be indicated.

 

Due to her history of rheumatoid arthritis, her calculated ASCVD risk should be multiplied by 1.5, yielding an ASCVD risk of 10.5% placing her in the intermediate risk category. Moderate intensity statin would be indicated. The ESC gives a Class IIa (LOE B) indication to multiply the calculated total CVD risk by a factor of 1.5 in adults with rheumatoid arthritis due to the observed 50% increased CVD risk in patients with rheumatoid arthritis.

 

This 50% increase in CVD risk attributed to RA is present beyond traditional risk factors, making answer choice C wrong.

 

Answer A is incorrect because when borderline risk is calculated, one should still look for risk enhancers that could potentially increase ASCVD risk before final determination of statin indication.

 

Answer choice D is false as there is no contraindication to take both methotrexate and statins together.

 

Note that it is appropriate to use the pool cohort equations and American risk thresholds for this patient since she is in America where the PCE was validated (versus using SCORE2 risk model which would be more appropriate for European populations).

Main Takeaway

Inflammatory conditions including rheumatoid arthritis and inflammatory bowel disease increase a person’s risk for ASCVD events. Specifically for rheumatoid arthritis, there is a Class IIa indication to multiply the calculated risk score by 1.5 to account for rheumatoid arthritis as a risk enhancer.

Guideline Loc.

Section 3.4.6

Ms. J is a 57-year-old woman with a past medical history of myocardial infarction resulting in ischemic cardiomyopathy, heart failure with reduced ejection fraction, and major depressive disorder who presents today for follow-up. She reports feeling extremely overwhelmed lately due to multiple life stressors. She is on appropriate cardiovascular GDMT agents and is not prescribed any medications for her mood disorder.

True or false: in addition to psychotherapy for stress management, it is appropriate to consider Ms. J for anti-depressant SSRI pharmacotherapy at this time to improve cardiovascular outcomes.

A

True

B

False

Explanation

The correct answer is FALSE.

An ESC class 3 recommendation states that SSRIs, SNRIs, and tricyclic antidepressants are not recommended in patients with heart failure and major depression; this is based on data suggesting potential lack of SSRI efficacy for reducing depression or cardiovascular events, as well as safety data indicating an association between SSRI use and increased risk of CV events and all-cause as well as cardiovascular mortality among HF patients. Mental health disorders are associated with worse outcomes in patients with ASCVD and appropriate treatment effectively reduces stress symptoms and improves quality of life. Nonpharmacologic modalities of treatment (exercise therapy, psychotherapy, collaborative care) should be considered before pharmacotherapy to improve cardiovascular outcomes in patients with heart failure.

Of note, the ESC suggests SSRI treatment be considered for patients with coronary heart disease (without HF) and moderate-to-severe major depression based on data that SSRI treatment is associated with lower rates of CHD readmission (RR 0.63), all-cause mortality (RR 0.56), and the composite endpoint of all-cause mortality/MI/PCI (HR 0.69) vs. no treatment. This is a class 2a recommendation.

ESC also gives a class 2a recommendation to consider referral to psychotherapeutic stress management for individuals with stress and ASCVD to improve CV outcomes and reduce stress symptoms.

The ACC/AHA guidelines do not provide focused recommendations regarding mental health considerations in patients with elevated cardiovascular risk.

Main Takeaway

It is important to consider mental health treatment in patients with ASCVD as mental disorders are associated with increased CVD risk and poor patient prognosis, and data support that mental health interventions can improve overall and CVD outcomes, as well as improve quality of life.

Guideline Loc.

Section 4.4

Mr. HC is a 50-year-old man presenting for a routine clinic visit. He is not sure the last time he had a lipid panel drawn, and would like one today, but ate lunch just prior to your appointment – a delicious plate of 50% fruits and vegetables, 25% lean meats, and 25% whole grains as you had previously recommended.

True or False: Mr. HC should return another day to obtain a fasting lipid panel.

TRUE

FALSE

Answer choices

TRUE

FALSE

Explanation 

This statement is False. A non-fasting lipid panel is appropriate for risk stratification and lipid evaluation in most patients per the ESC guidelines.

While no level of evidence in provided in the ESC guidelines, this recommendation is consistent with AHA/ACC cholesterol guidelines, which have also largely moved away from fasting lipid panels for most patients and give a Class 1 (LOE B) recommendation to obtaining a fasting or nonfasting plasma lipid profile for ASCVD estimation and baseline LDL-C in adults 20 years of age or older.

The ESC recommendation is based upon large trials showing that results of fasting and non-fasting panels are largely similar. This is similar to the AHA/ACC guidelines, which note non-fasting and fasting LDL-C change minimal over time following a normal meal, while HDL-C and tryiglycerides appear to have similar prognostic significance with cardiovascular outcomes in fasting or nonfasting states.

A fasting lipid panel should be considered in those with hypertriglyceridemia, metabolic syndrome, and diabetes mellitus, as consumption of food or drink can have direct and immediate effects on TG and blood glucose values.

Main Takeaway

A non-fasting lipid panel is appropriate for the majority of patients undergoing lipid evaluation and cardiovascular risk stratification. 

Guideline Loc.

Section 4.6.1

An asymptomatic 55-year-old man with no past medical history presents to clinic after having a cardiac CT as part of an executive physical. His coronary artery calcium (CAC) score was 200 and the coronary CTA demonstrated isolated 70% stenosis of the left circumflex coronary artery. He is asymptomatic and able to jog 2 miles daily without limitation. He was recently started on aspirin 81 mg daily and atorvastatin 40 mg daily by his primary care provider. His LDL is 50 mg/dL, HbA1c is 6.0%. His BP is 108/70. What would you recommend?

A

Stop aspirin 81 mg daily as he has not had an ASCVD event or revascularization

B

Cardiac catheterization and stent placement in the left circumflex

C

Increase atorvastatin to 80 mg daily

D

Stress test

E

No change in management

Answer choices

A

Stop aspirin 81 mg daily as he has not had an ASCVD event or revascularization

B

Cardiac catheterization and stent placement in the left circumflex

C

Increase atorvastatin to 80 mg daily

D

Stress test

E

No change in management

Explanation

The correct answer is E – no change in management.

Though the patient has not had an ASCVD event or revascularization, low-dose aspirin may be considered with definite evidence of CAD on imaging (Class IIb, LOE C).

He is asymptomatic and does not have high risk anatomy on CT (i.e., proximal LAD, left main disease, multivessel disease), so percutaneous coronary intervention or stress testing are not indicated.

His LDL is well controlled, so increasing atorvastatin would not be appropriate at this time.

Main Takeaway

Aspirin 75-100 md daily may be considered in the absence of MI or revascularization when there is definitive evidence of CAD on imaging (Class IIb, LOE C).

Guideline Loc.

Section 6.1

A 65-year-old man with a history of ischemic stroke 6 months ago presents to cardiology clinic to establish care. An event monitor was negative for atrial fibrillation and TTE with agitated saline study was negative for a patent foramen ovale. Therefore, his ischemic stroke was presumed to be non-cardioembolic in origin. He is currently taking lisinopril 5 mg daily for hypertension (BP in clinic is 115/70) and atorvastatin 40 mg daily. He has no history of significant gastrointestinal or other bleeding. What do you recommend next?

A

Start apixaban 5 mg BID

B

Increase lisinopril to 10 mg daily

C

Start aspirin 81 mg daily

D

Start aspirin 81 mg daily and clopidogrel 75 mg daily

E

Start aspirin 81 mg daily and ticagrelor 90 mg BID

Explanation

The correct answer is C – start aspirin 81mg daily.

For the secondary prevention of non-cardioembolic ischemic stroke or TIA, anti-platelet therapy is recommended with aspirin only (75-150 mg/day), dipyridamole + aspirin (slightly superior to aspirin), or clopidogrel alone (slightly superior to aspirin) (Class I, LOE A).

DAPT with aspirin and clopidogrel or aspirin and ticagrelor should be considered in the immediate period after a minor ischemic stroke or TIA (3 weeks after event, Class IIa), but not 6 months after an ischemic stroke. Dual antiplatelet therapy with aspirin and clopidogrel increases bleeding risk without a significant benefit over either agent alone. Dual antiplatelet therapy with aspirin and ticagrelor increases bleeding risk, but does not improve disability incidence.

Oral anticoagulation would be recommended for a cardioembolic stroke, which does not fit the clinical picture.

His BP is well controlled so increasing lisinopril is not necessary.

Main Takeaway

For the secondary prevention of an ischemic stroke or TIA, anti-platelet therapy with aspirin, aspirin + dipyridamole, or clopidogrel alone is recommended.

Guideline Loc.

6.3, page 3296-3297

Please choose the CORRECT statement from the ones below.

A

CAC scoring can be considered to improve ASCVD risk classification around treatment decision thresholds.

B

Patients with type 1 or type 2 diabetes are considered very high CV risk, regardless of comorbidities and other risk factors.

C

CKD does not increase the cardiovascular risk in the absence of other risk factors.

D

Men and women older than 65 years old are at high cardiovascular risk.

Explanation

Option A is correct. Coronary artery calcium (CAC) scoring can reclassify CVD risk upwards and downwards in addition to conventional risk factors and may thus be considered in men and women with calculated risks around decision thresholds (Class IIb, Level B). If CAC is detected, its extent should be compared with what would be expected for a patient of the same sex and age. CAC scoring does not provide direct information on total plaque burden or stenosis severity and can be low or even zero in middle-aged patients with soft non-calcified plaque.

Option B is false. Not all patients with diabetes are very high risk by default.

·       Moderate risk: well controlled diabetes, <10 years duration, without evidence of target organ damage and no additional ASCVD risk factors.

·       High risk: patients not fulfilling the criteria above, without ASCVD and/or severe target organ damage.

·       Very high risk: diabetic patients with established ASCVD and/or severe target organ damage.

Severe target organ damage is defined by:

·       eGFR <45 mL/min/1.73 m2

·       eGFR 45-59 mL/min/1.73 m2 and microalbuminuria (albumin-to-creatinine ratio, ACR 30 -300 mg/g)

·       Proteinuria (ACR >300 mg/g)

·       Presence of microvascular disease in at least 3 different sites (e.g., microalbuminuria + retinopathy + neuropathy

Option C is false. CKD carries at least a high CVD risk even in the absence of diabetes or ASCVD.

·       Moderate CKD carries a high CVD risk:

o   eGFR 30−44 mL/min/1.73 m2 and ACR <30

o   eGFR 45−59 mL/min/1.73 m2 and ACR 30−300

o   eGFR ≥60 mL/min/1.73 m2 and ACR >300

·       Severe CKD carries a very high CVD risk:

o   eGFR<30 mL/min/1.73 m2

o   eGFR 30−44 mL/min/1.73 m2 and ACR >30

 

Option D is false. There is an age difference between men and women with regards to cardiovascular risk. Age is a major CVD risk driver, but age cutoffs should be used with flexibility.

·       Women < 50 years-old and men < 40 years old are usually at low 10-year CVD risk. It is still important to be aware of unfavorable modifiable risk factors that can sharply increase their lifetime CVD risk.

·       Women > 75 years-old and men > 65 years-old are usually at high 10-year CVD risk.

·       Only between the ages of 55 – 75 years in women and 40 – 65 years in men does the 10-year CVD risk vary around commonly used thresholds for intervention.

 

Of note:

·       In younger, apparently healthy patients, we also discuss lifetime CVD risk estimates since 10-year risk assessments often underestimate risk.

·       In an aging population, treatment decisions should take competing non-CVD risk into account.

·       In patients with established ASCVD we can discuss about residual CVD – defined as the risk estimated after initial lifestyle changes and risk factor treatment.

Main Takeaway

Estimating CVD risk is not only important in apparently healthy patients, but also in patients with diabetes, renal disease, established ASCVD, or older patients. This can provide information to allow shared decision making and personalized approach for our patients.

Guideline Loc.

Table 3, page 3237; Section 3.2.3., page 3243; Table 4, page 3244

Ms. Priya Clampsia is a 58-year-old never-smoker with a history of hypertension. Her BMI is 29 kg/m2. She also mentions having pre-eclampsia during her pregnancy many years ago. She describes a predominately sedentary lifestyle and works as a receptionist. You see her in the clinic to discuss routine preventive care. Her most recent lipid panel results were LDL of 101 mg/dL, HDL of 45 mg/dL, and triglycerides of 190 mg/dL. What additional step will provide valuable information regarding her CVD risk profile?

A

Send additional lab workup including C-reactive protein and lipoprotein (a)

B

Measure her waist circumference

C

Assess her work stress

D

Ask her about history of preterm birth

E

B, C, and D

Explanation

The correct answer is E – measuring her waist circumference, assessing her occupational stress, and obtaining history about adverse pregnancy outcomes including preterm birth all add valuable information for CVD risk stratification.

BMI is easily measured and can be used to define categories of body weight. However, body fat stores in visceral tissue carry higher risk than subcutaneous fat and therefore, waist circumference can be a simple way to measure global and abdominal fat. When waist circumference is ≥102 cm in men and ≥88 cm in women, weight reduction is advised. While these WHO thresholds are widely accepted in Europe, it is important to note that different cut-offs may be appropriate in different ethnic groups.

Work stress is important to ascertain as well because there is preliminary evidence of the detrimental impact of worse stress on ASCVD health, independent of conventional risk factors and their treatment. Work stress is determined by job strain (i.e., the combination of high demands and low control at work) and effort-reward imbalance.

Pre-eclampsia is associated with increase in CVD risk by factor of 1.5-2.7 compared with all women. Both preterm (RR 1.6) and still birth (RR 1.5) are also associated with a moderate increase in CVD risk. Taking a thorough pregnancy history is important in determining future cardiovascular risk in women. The ESC guidelines give a Class IIb (LOE B) recommendation that in women with history of premature or stillbirth, periodic screening for hypertension and DM may be considered. Of note, the 2018 ACC/AHA guidelines include preeclampsia and premature menopause (occurring at age <40 years) as risk-enhancing factors for statin therapy but state that the mechanism or cause of preterm birth is often unknown, so it is difficult to include it as a risk-enhancing factor.

Choice A (sending additional lab workup including CRP and LPa) is incorrect. The ESC guidelines do not recommend using routine circulating biomarkers as they do not improve risk prediction and publication bias distorts the evidence (Class III, LOE B). While some biomarkers like lipoprotein (a) are promising, further work is still needed. Conversely, the 2019 ACC/AHA guidelines do include, if measured, elevated high-sensitivity C-reactive protein (≥2mg/L) and elevated Lp(a) (>50mg/dL or >125nmol/L) and elevated apoB (≥130 mg/dL) as risk-enhancing factors. Specific indications for measuring Lp(a) include family history of premature ASCVD and specific indications for measuring apoB include triglyceride ≥200mg/dL.

Main Takeaway

The ESC guidelines do not recommend routine measurement of additional circulating and urine biomarkers as further data and research is still needed in this area; however, there are specific situations in which these biomarkers may be warranted.

Guideline Loc.

Section 3.3.7, 3.3.9, 3.3.10, 3.4.12

Mr. O is a 48-year-old man with a past medical history significant for obesity (BMI is 42kg/m2), hypertension, type 2 diabetes mellitus, and hypercholesterolemia. His calculated ASCVD risk score today is 18.8%. You counsel him on the importance of weight loss in the context of CVD risk reduction. Which of the following weight loss recommendations is appropriate?

A

Maintaining a weight loss of at least 25% from baseline is required to influence blood pressure, cholesterol, and glycemic control. 

B

Hypocaloric diets lead to short term weight loss, but a healthy diet should be maintained over time to reduce CVD risk.

C

Liraglutide can be used to induce weight loss, as an alternative to diet and exercise.

D

Bariatric surgery is effective for weight loss but has no ASCVD risk reduction benefit.

Explanation

The correct answer is B. Energy restriction is the cornerstone of management of obesity. All the different types of hypocaloric diets achieve similar short-term weight loss, but these effects tend to diminish by 12 months. It is a class I recommendation to maintain a healthy diet over time to achieve CVD risk reduction. The Mediterranean diet is an example of a diet that can have persistent CV benefit beyond the 12 months.

Choice A is incorrect because maintaining even a moderate weight loss of 5 – 10% from baseline has favorable effects on risk factors including blood pressure, cholesterol, and glycemic control, as well as on premature all-cause mortality.

Choice C is incorrect because medications approved as aids to weight loss (such as liraglutide, orlistat and naltrexone/bupropion) may be used in addition to lifestyle measures to achieve weight loss and maintenance; they are not alternatives to a healthy lifestyle. Meta-analysis of medication-assisted weight loss found favorable effects on BP, glycemic control, and ASCVD mortality.

Choice D is incorrect because patients undergoing bariatric surgery had over 50% lower risks of total ASCVD and cancer mortality compared with people of similar weight who did not have surgery. Bariatric surgery should be considered for obese high-risk individuals when lifestyle change does not result in maintained weight loss (Class IIa). The ACC/AHA guidelines focused primarily on lifestyle interventions for obesity and had no specific recommendations for bariatric surgery or medication-assisted weight loss.

Main Takeaway

Weight reduction (even as low as 5-10% from baseline) and long-term maintenance of a healthy diet are recommended to improve the CVD risk profile of overweight and obese people. Medication and/or bariatric surgery may have a useful adjunctive role in some patients.

Guideline Loc.

Section 4.3.3

Mr. A. C. is a 78-year-old gentleman with a long-standing history of HTN receiving antihypertensive medications & dietary management for blood pressure control. What is the target diastolic blood pressure recommendation for all treated patients such as Mr. A.C.?

A

< 80 mmHg

B

< 90 mmHg

C

< 70 mmHg

D

< 95 mmHg

E

< 100 mmHg

Explanation

 

The correct answer is A: DBP < 80 mmHg

Blood pressure treatment targets: when drug treatment is used, the aim is to control BP to target within 3 months. Blood pressure treatment targets in the 2021 ESC Prevention guidelines are more aggressive than previously recommended, as evidence now suggests the previously recommended targets were too conservative, especially for older patients.

The magnitude of BP lowering is the most important driver of benefit.

·       It is recommended that the first objective of treatment is to lower BP to <140/90 mmHg in all patients, and that subsequent BP targets are tailored to age and specific comorbidities (Class 1).

·       In treated patients aged 18-69 years, it is recommended that SBP should ultimately be lowered to a target range of 120 – 130 mmHg in most patients (Class 1).

·       In treated patients aged ≥70 years, it is recommended that SBP should generally be targeted to <140 and down to 130 mmHg if tolerated (Class 1).

·       In all treated patients, DBP is recommended to be lowered to <80 mmHg (Class I).

This change in the BP target range for older people compared with the 2016 ESC prevention guidelines is supported by evidence that these treatment targets are safely achieved in many older patients and are associated with significant reductions in the risk of major stroke, HF, and CV death. It also takes into account that the even lower SBP in the intensively treated group in SPRINT (Systolic Blood Pressure Intervention Trial) (mean 124 mmHg) probably reflects a conventional office SBP range of 130-139 mmHg. It is recognized, however, that the evidence supporting more strict targets is less strong for very old people (>80 years) and those who are frail. Also, in these older and especially frail patients, it may be difficult to achieve the recommended target BP range due to poor tolerability or adverse effects, and high-quality measurement and monitoring for tolerability and adverse effects is especially important in these groups.

Main Takeaway

The first step in HTN management in all groups is a reduction to SBP < 140 mmHg and DBP < 80 mmHg, with further targets depending on age and comorbidities as specified by Table 18 of the 2021 ESC Prevention Guidelines.

Guideline Loc.

1.     4.7.5.3 page 3285

2.     Table 18 page 3287

What percentage of the European population currently meets the recommended physical activity guidelines (150 minutes moderate-intensity activity weekly or 75 minutes vigorous-intensity activity weekly)?

A

<10%

B

10-25%

C

25-50%

D

50-75%

E

>75%

Explanation

The correct answer is A: <10% of the European population currently meets the recommended physical activity guidelines.

The American Heart Association, European Society of Cardiology, and World Health Organization all share the recommendation that adults should engage in 150 minutes per week of moderate-intensity physical activity or 75 minutes per week of vigorous-intensity activity. They recognize that additional health benefits may be garnered from incremental increases to 300 minutes per week of moderate intensity activity or 150 minutes per week of vigorous intensity activity, with a recommendation to include both aerobic and muscular strength training activities.

According to the WHO, physical inactivity is the 4th leading cause of death in the world. The statistics regarding physical inactivity are staggering. Recent studies have shown that <10% of the European population meets the minimum recommended levels of physical activity. Similarly, ¼ adults and ¾ adolescents (aged 11-17) do not currently meet the global recommendations for physical activity. The World Health Organization has created a Global Action Plan on Physical Activity 2018-2030 with the goal to achieve a 15% relative reduction in the global prevalence of physical inactivity by 2030.

Society level interventions to increase physical activity have been proposed including school-based activity programs, improved accessibility of exercise facilities across the socioeconomic spectrum, and governmental consideration of physical activity when designing cities (i.e. including pedestrian and cycling lanes). Other policy suggestions with varying levels of evidence include focused media campaigns, economic incentives, targeting labeling of physical activity opportunities, and work-place wellness programs.

Main Takeaway

Despite growing awareness of the health consequences of sedentary behavior, fewer than 10% of adults currently meet the minimum recommended quantity of physical activity. Public health leaders may continue to consider novel legislative initiatives to augment physical activity on a societal level with architectural design and financial incentives.

Guideline Loc.

Section 5.2

A 65-year-old woman with a history of hypertension, type 2 diabetes mellitus, and coronary artery disease with remote PCI to the RCA presents for follow-up. She has stable angina symptoms that are well controlled with metoprolol tartrate 25 mg BID and are not lifestyle limiting. She takes aspirin 81 mg daily and atorvastatin 40 mg daily. Her LDL-C is 70 mg/dL, hemoglobin A1c is 7.0%, and eGFR is >60. In clinic, her BP is 118/80 mmHg. What is the next step in management?

A

Increase atorvastatin for goal LDL-C < 55 mg/dL

B

No change in management

C

Add isosorbide mononitrate 30 mg daily

D

Stop aspirin

E

Start a sulfonylurea

Explanation

The correct answer is A – increase atorvastatin for goal LDL-C < 55 mg/dL.

In patients with established ASCVD, the ESC guidelines advocate for an LDL goal of < 55 mg/dL with at least a 50% reduction from baseline levels (Class I, LOE A). This patient has stable angina which is not lifestyle limiting; as such, further anti-anginal therapy is not necessary. She has known CAD with prior PCI, so aspirin therapy is appropriate for secondary prevention (Class I, LOE A). There is no indication for a sulfonylurea as her diabetes is well controlled. Notably, in persons with type 2 DM and ASCVD, the use of a GLP-1RA or SGLT2 inhibitor with proven outcome benefits is recommended to reduce CV and/or cardiorenal outcomes (Class I, LOE A).

Main Takeaway

For people with established ASCVD, the ESC-recommended LDL-C goal is < 55 mg/dL with a goal reduction of at least 50%.

Guideline Loc.

Section 6.1

The 2021 ESC CV Prevention guidelines recommend a stepwise approach to risk stratification and treatment options. What is the first step in risk factor treatment regardless of past medical history, risk factors, or established ASCVD?

A

Initiate statin for goal LDL <100 mg/dl

B

Assess family history of premature CVD

C

Counsel on ketogenic diet

D

Stop smoking and lifestyle recommendations

Explanation

The correct answer is D – stop smoking and lifestyle recommendations. Smoking cessation and lifestyle modifications are recommended for everyone across the spectrum of ASCVD risk including for both primary and secondary prevention (Class 1). It is worth noting that many patients can move themselves towards a lower risk category without taking drugs just by stopping smoking.

Option A is incorrect. While initiating statin therapy for goal LDL <100 mg/dL may be an appropriate treatment option for some patients, it is not the first step per the “stepwise approach” recommended in the ESC guidelines. Whether or not to initiate a statin depends on a multitude of factors including estimated 10-year CVD risk, age, comorbidities, established ASCVD, and patient preference. The first step for patients with established ASCVD includes LDL-C reduction to goal <70 mg/dL (class I) with intensification to a goal LDL-C <55mg/dL based on residual 10-year CVD risk, lifetime CVD risk and treatment benefit, comorbidities, frailty, and patient preference.  Primary prevention of ASCVD first targets LDL-C goal <100 (class IIa) in appropriately selected patients.

Option B is incorrect. While assessing family history of premature CVD should be part of an initial evaluation and certainly considered a risk enhancing factor, it is not a modifiable risk factor with regards to treatment.

Option C is incorrect. A ketogenic diet is not endorsed nor recommended by ACC/AHA or ESC.

Studies have shown the benefit of a stepwise approach to treatment intensification. Attainment of treatment goals is similar, side effects are fewer, and patient satisfaction is significantly greater with such an approach. It is not recommended to stop assessment of treatment goals after the first step.

Main Takeaway

A stepwise approach to treatment intensification is recommended. The first steps for all patients are to stop smoking and institute lifestyle recommendations.

Guideline Loc.

·       3.2.3 Figures 6-8

Mr. Daniel Collins is a 58-year-old man with hypertension, chronic kidney disease (CKD), and obesity who presents to your clinic for a routine physical examination. Vitals are as follows: BP 143/79 mmHg, HR 89 bpm, O2 99% on room air, weight 106 kg, BMI 34.5 kg/m2. Recent laboratory testing revealed: creatinine 1.24 mg/dL, total cholesterol 203 mg/dL, HDL 39 mg/dL, LDL 112 mg/dL, TG 262 mg/dL. His current medications include lisinopril and rosuvastatin. You recommend increasing the dose of lisinopril to treat uncontrolled hypertension. What additional step(s) are indicated at this visit?

A

Order urine albumin-to-creatinine ratio

B

Ask the patient how often they have been bothered by trouble falling or staying asleep, or sleeping too much

C

Perform depression screening

D

All of the above

Explanation

 

The correct answer is D – all of the above.

Answer A is correct. The ESC gives a Class I (LOE C) indication that all CKD patients, with or without diabetes, should undergo appropriate screening for ASCVD and kidney disease progression, including monitoring for changes in albuminuria. Cardiovascular disease is the leading cause of morbidity and death among patients with CKD. Even after adjusting for risk factors, including diabetes and hypertension, there is a linear increase in CV mortality with decreasing GFR below ~60-75 mm/min/1.73m2. Specific CKD-related risk factors include uremia-mediated inflammation, oxidative stress, and vascular calcification.

Answer choice B is also correct. In patients with ASCVD, obesity, and hypertension, the ESC gives a Class I (LOE C) indication to regularly screen for non-restorative sleep by asking the question related to sleep quality as follows: “‘How often have you been bothered by trouble falling or staying asleep or sleeping too much?”. Additionally, if there are significant sleep problems that are not responding within four weeks to improving sleep hygiene, referral to a specialist is recommended (Class I, LOE C). However, despite the strong association of OSA with CVD, including hypertension, stroke, heart failure, CAD, and atrial fibrillation, treatment of OSA with CPAP has failed to improve hard CVD outcomes in patients with established CVD. Interventions that focus on risk factor modification, including reduction of obesity, alcohol intake, stress, and improvement of sleep hygiene, are important.

Answer choice C is also correct. The ESC gives a Class I (LOE C) recommendation that mental health disorders with either significant functional impairment or decreased use of healthcare systems be considered as influencing total CVD risk. All mental disorders are associated with the development of CVD and reduced life expectancy. Additionally, the onset of CVD is associated with an approximately 2-3x increased risk of mental health disorders compared to a healthy population. As such, screening for mental health disorders should be performed at every consultation (2-4x/year).

Main Takeaway

In addition to traditional ASCVD risk factors, other clinical conditions, including sleep apnea, CKD, and mental health conditions, are important to screen for and treat if present.

Guideline Loc.

Sections 3.4.1, 3.4.9, 3.4.10

Mr. V is a 37-year-old man who presents to clinic after a recent admission for anterior STEMI and is status-post emergent percutaneous intervention to the proximal LAD. He has mixed hyperlipidemia and a 10 pack-year history of (current) tobacco smoking. Which of the following points related to tobacco use is LEAST appropriate for today’s visit?

A

Providing assessment and encouragement for smoking cessation, even if for only a 30-second “very brief advice” intervention.

B

Reviewing and offering pharmacotherapy support options for smoking cessation if Mr. V expresses readiness to quit today.

C

Recommending a switch from traditional cigarettes to e-cigarettes as a first step towards cessation, as e-cigarettes are safer for use.

D

Discussing that smoking cessation is strongly recommended for all patients, regardless of potential weight gain.

Explanation

Answer C is LEAST appropriate and therefore is the correct answer.

Answer C is not appropriate. Although e-cigarettes may be more effective than nicotine replacement therapy (NRT) for smoking cessation, the long-term effects of e-cigarettes on cardiovascular and pulmonary health are unknown. According to the 2019 ACC/AHA prevention guidelines, e-cigarettes may increase the risk of CV and pulmonary diseases; their use has been reportedly associated with arrhythmias and hypertension. Therefore, neither the ESC nor ACC/AHA suggest clinicians recommend e-cigarettes over traditional cigarettes to patients.

Answer A: Smoking cessation is one of the most effective CVD risk-lowering preventive measures, with significant reductions in (repeat) myocardial infarctions or death. ESC guidelines emphasize the importance of encouraging smoking cessation even in settings where time is limited. “Very brief advice” on smoking is a proven 30-second clinical intervention, developed in the UK, which identifies smokers, advises them on the best method of quitting, and supports subsequent quit attempts. While ESC does not explicitly suggest a frequency of assessment, the 2019 ACC/AHA guidelines specifically recommend that “all adults should be assessed at every healthcare visit for tobacco use and their tobacco use status recorded as a vital sign to facilitate tobacco cessation.”

Answer B: The ESC suggests (class 2) that offering follow-up support, nicotine replacement therapy, varenicline, and bupropion individually or in combination should be considered in smokers. A meta-analysis of RCTs in patients with ASCVD reflects that varenicline (RR 2.6), bupropion (RR 1.4), telephone therapy (RR 1.5), and individual counselling (RR 1.6) all increased quit rates versus placebo; NRT therapies were well-tolerated but had inconclusive effects on quit rates (RR 1.22 with 95% CI 0.72-2.06). The 2019 ACC/AHA recommendation to combine behavioral and pharmacotherapy interventions to maximize quit rates is a class 1 recommendation.

Answer D: The ESC gives a class 1 recommendation to recommending smoking cessation regardless of weight grain. Smokers who quit may expect an average weight gain of 5 kg, but the health benefits of tobacco cessation (both CVD and non-CVD related) consistently outweigh risks from weight gain. Weight gain does not lessen the ASCVD benefits of cessation. The 2019 ACC/AHA guidelines do not specifically comment on weight considerations with smoking cessation.

Main Takeaway

Stopping smoking is potentially the most effective of all preventive measures. All smoking of tobacco should be stopped, as tobacco use is strongly and independently causal of ASCVD (Class 1). Smoking cessation should be regularly assessed for and encouraged, and pharmacotherapy and follow-up support for cessation should be considered for patients who are ready for a quit attempt.

Guideline Loc.

Section 4.5, Table 9

Ms. BW presents after her best friend was diagnosed with hypertension and is interested in measuring her own blood pressure.

 

According to the ESC Guidelines, what BP screening approach is recommended for making a diagnosis of hypertension?

 

A

Repeated measurements in one visit

B

A single measurement in a single visit

C

Repeated measurements in more than one visit  

D

Reported patient history 

Explanation 

The correct answer is C – Repeated measurements in more than one visit.

It is recommended to base the diagnosis of hypertension on repeated office BP measurements on more than one visit except when hypertension is severe (e.g., Grade 3—defined as SBP > 180 and/ or DBP >110mmHg—and especially in high-risk patients) (Class I, LOE C). In addition to recommending repeat measurements across visits, the guidelines provide a number of considerations for appropriately measuring blood pressure, such as taking measurements when seated in a quiet environment for 5 minutes and measuring in both arms at the first visit and using the higher-level value arm for visits thereafter (see Table 14 on page 3283).

Additionally, home blood pressure monitoring is recommended as an alternative to repeated office measurements. Blood pressure measurements are taken with a semiautomated, validated cuff for 3 consecutive days – and 6-7 days being preferred – in the morning and at night, averaged over that period. Notably, home blood pressure thresholds for the diagnosis of hypertension are lower than for that of in-office measurements, with a daytime systolic of 135mmHg or diastolic of 85mmHg given as the level at which hypertension is diagnosed, as opposed to 140mmHg and 90mmHg for systolic and diastolic levels, respectively, given for in-office diagnosis.

Main Takeaway

With the exception of those with severely elevated blood pressures, the diagnosis of hypertension requires repeated measurements across multiple office visits.

Guideline Loc.

Sections 4.7.1 and 4.7.2, Table 13 and 14, Figure 14

In patients with a low risk of cardiovascular disease, which of the following is true?

A

Aspirin does not affect the risk of ischemic stroke

B

Aspirin increases the risk of fatal bleeding.

C

Aspirin reduces the risk of non-fatal MI.

D

Aspirin reduces cardiovascular mortality

Explanation

In 2019, an updated meta-analysis of aspirin for primary prevention of cardiovascular events found that patients with a low risk of CVD taking aspirin did not have a reduction in all-cause or cardiovascular mortality. There was a lower risk of non-fatal MI (RR 0.82) and ischemic stroke (RR 0.87). However, aspirin was also associated with a  higher risk of major bleeding (RR 1.50), intracranial bleeding (RR 1.32), and major GI bleeding (RR 1.52). There was no difference in the risk of fatal bleeding (RR 1.09).

Accordingly, the ESC does not recommend antiplatelet therapy in individuals with low/moderate CV risk due to the increased risk of major bleeding (Class III, LOE A).

Although aspirin should not be given routinely to patients without established ASCVD, we cannot exclude that in some patients at high or very high CVD risk, the benefits may outweigh the risks.

Main Takeaway

In patients with low/moderate risk of CVD, aspirin for primary prevention is not recommended due to the higher risk of bleeding. For those at higher risk of CVD, low-dose aspirin may be considered for prevention in the absence of contraindications.

Guideline Loc.

Section 4.9.1, Page 3291